Sunteți pe pagina 1din 94

Universidad de El Salvador

Facultad de Ciencias Naturales y Matemática


Escuela de Matemática
Licenciatura en Matemática y
Licenciatura en Estadı́stica
GEOMETRÍA I
Equipo de Diseño:
Nahomy Jhopselyn Hernández Cruz,
Gabriel Alexander Chicas Reyes,
Eduardo Arnoldo Aguilar Cañas,
Héctor Enmanuel Alberti Arroyo,
Humberto Alfonso Sermeño Villalta,
Ernesto Américo Hidalgo Castellanos,
Juan Agustı́n Cuadra,
Claudia Patricia Corcio López de Beltrán,
Carlos Mauricio Canjura Linares,
Oscar Armando Hernández Morales,
Aarón Ernesto Ramı́rez Flores,
Manuel Alejandro Mundo Dueñas,
Francisco Asdrubal Hernández Ramı́rez

Resumen
En el presente apunte se examinan los conceptos geométricos básicos, y a partir de estos
se construye el modelo denominado Geometrı́a Euclidiana. Las temáticas se desarrollan de la
forma más elemental y tienen una base importante en la observación y la construcción.
Se trata de iniciar a los estudiantes en el fascinante mundo de la Geometrı́a, a través de la
axiomatización de la misma, con el ingrediente adicional que tiene esta área de la matemática:
el uso de la creatividad y la imaginación.
El estudio de la Geometrı́a y especı́ficamente de este modelo es importante, porque en pri-
mer lugar, desarrolla habilidades vitales para cualquier estudiante de matemática y ciencias,
tales como el descubrimiento de patrones, invariantes y el desarrollo de una lógica aguda; en
segundo lugar, porque se rescata lo que históricamente fue el nacimiento de la Matemática
como un sistema lógico riguroso.
Por otro lado, este documento es un apunte de clase de Geometrı́a Plana que se estudian
en la asignatura Geometrı́a I, desde los conceptos fundamentales y las propiedades básicas

1
Geometrı́a I.

de los triángulos, cuadriláteros y circunferencias, hasta el estudio detallado de las rectas y


puntos notables de un triángulo. Además, se estudian tanto problemas métricos como de in-
cidencia: medidas de segmentos, ángulos, áreas, paralelismo, perpendicularidad, concurrencia
y colinealidad. Y, ha sido elaborado con la colaboración de ex alumnos del Programa Jóvenes
Talento y docentes de la escuela de Matemática. Esta recopilación de información extraı́da
de distintos libros, cubre la totalidad de los contenidos exigidos en el programa de estudio.

Figura 1: Porisma de Steiner

UES-FCNM-Escuela de Matemática-Licenciatura en Matemática y Licenciatura en Estadı́stica


pág. 2
Geometrı́a I. ÍNDICE

Índice
1. Aspectos históricos 5

2. Segmentos 8
2.1. Introducción . . . . . . . . . . . . . . . . . . . . . . . . . . . . . . . . . . . . . . . . 8
2.2. Lı́neas . . . . . . . . . . . . . . . . . . . . . . . . . . . . . . . . . . . . . . . . . . . 8
2.3. Rayos . . . . . . . . . . . . . . . . . . . . . . . . . . . . . . . . . . . . . . . . . . . 9
2.4. Segmento de recta . . . . . . . . . . . . . . . . . . . . . . . . . . . . . . . . . . . . 9
2.5. Punto medio de un segmento . . . . . . . . . . . . . . . . . . . . . . . . . . . . . . . 9
2.6. Operaciones con segmentos colineales . . . . . . . . . . . . . . . . . . . . . . . . . . 10
2.7. Ejercicios . . . . . . . . . . . . . . . . . . . . . . . . . . . . . . . . . . . . . . . . . 12
2.8. Ejercicios . . . . . . . . . . . . . . . . . . . . . . . . . . . . . . . . . . . . . . . . . 13

3. Ángulos 15
3.1. Ángulos formados por dos rectas y una trasversal a ellas. . . . . . . . . . . . . . . . 16
3.2. Problemas . . . . . . . . . . . . . . . . . . . . . . . . . . . . . . . . . . . . . . . . . 18

4. triángulos 21
4.1. Clasificación de Triángulos . . . . . . . . . . . . . . . . . . . . . . . . . . . . . . . . 21
4.2. Teoremas Fundamentales del Triángulo . . . . . . . . . . . . . . . . . . . . . . . . . 22
4.3. Perpendicularidad . . . . . . . . . . . . . . . . . . . . . . . . . . . . . . . . . . . . . 23
4.4. Rectas Notables de un triángulo . . . . . . . . . . . . . . . . . . . . . . . . . . . . . 25
4.5. Ejercicios . . . . . . . . . . . . . . . . . . . . . . . . . . . . . . . . . . . . . . . . . 25
4.6. Congruencia de Triángulos. . . . . . . . . . . . . . . . . . . . . . . . . . . . . . . . 30
4.6.1. Criterios de Congruencia de triángulos . . . . . . . . . . . . . . . . . . . . . 31
4.6.2. Teorema de la Base Media . . . . . . . . . . . . . . . . . . . . . . . . . . . . 32
4.6.3. Ejercicios . . . . . . . . . . . . . . . . . . . . . . . . . . . . . . . . . . . . . 33

5. Cuadriláteros 39
5.1. Clasificación de Cuadriláteros (De acuerdo a sus diagonales . . . . . . . . . . . . . . 39
5.2. Paralelogramos . . . . . . . . . . . . . . . . . . . . . . . . . . . . . . . . . . . . . . 40
5.3. Rectángulos . . . . . . . . . . . . . . . . . . . . . . . . . . . . . . . . . . . . . . . . 41
5.4. Rombos . . . . . . . . . . . . . . . . . . . . . . . . . . . . . . . . . . . . . . . . . . 42
5.5. Trapezoides . . . . . . . . . . . . . . . . . . . . . . . . . . . . . . . . . . . . . . . . 43
5.6. Trapecios . . . . . . . . . . . . . . . . . . . . . . . . . . . . . . . . . . . . . . . . . 43
5.7. Problemas . . . . . . . . . . . . . . . . . . . . . . . . . . . . . . . . . . . . . . . . . 45

6. La Circunferencia 49
6.1. Elementos de la circunferencia . . . . . . . . . . . . . . . . . . . . . . . . . . . . . . 49
6.2. Ángulos en la circunferencia . . . . . . . . . . . . . . . . . . . . . . . . . . . . . . . 49
6.3. Cuadriláteros Concı́clicos . . . . . . . . . . . . . . . . . . . . . . . . . . . . . . . . . 51
6.4. Rectas y Circunferencias tangentes a una circunferencia . . . . . . . . . . . . . . . . 53
6.5. Problemas . . . . . . . . . . . . . . . . . . . . . . . . . . . . . . . . . . . . . . . . . 55

UES-FCNM-Escuela de Matemática-Licenciatura en Matemática y Licenciatura en Estadı́stica


pág. 3
Geometrı́a I. ÍNDICE

7. Semejanza de Triángulos. 65
7.1. Proporcionalidad . . . . . . . . . . . . . . . . . . . . . . . . . . . . . . . . . . . . . 65
7.2. Teorema de Thales . . . . . . . . . . . . . . . . . . . . . . . . . . . . . . . . . . . . 66
7.3. Criterios de Semejanza de Triángulos . . . . . . . . . . . . . . . . . . . . . . . . . . 69
7.4. Potencia de Punto . . . . . . . . . . . . . . . . . . . . . . . . . . . . . . . . . . . . 70
7.5. Problemas . . . . . . . . . . . . . . . . . . . . . . . . . . . . . . . . . . . . . . . . . 72
7.6. Puntos y Rectas Notables del Triángulo. . . . . . . . . . . . . . . . . . . . . . . . . 81
7.6.1. Medianas . . . . . . . . . . . . . . . . . . . . . . . . . . . . . . . . . . . . . 81
7.6.2. Mediatrices . . . . . . . . . . . . . . . . . . . . . . . . . . . . . . . . . . . . 81
7.6.3. Alturas . . . . . . . . . . . . . . . . . . . . . . . . . . . . . . . . . . . . . . 82
7.6.4. Bisectrices . . . . . . . . . . . . . . . . . . . . . . . . . . . . . . . . . . . . . 83
7.6.5. Problemas . . . . . . . . . . . . . . . . . . . . . . . . . . . . . . . . . . . . . 84

UES-FCNM-Escuela de Matemática-Licenciatura en Matemática y Licenciatura en Estadı́stica


pág. 4
Geometrı́a I. 1 Aspectos históricos

1. Aspectos históricos
Euclides y Arquı́medes son las dos figuras más importantes de la Matemática griega. Mientras
que Arquı́medes es el investigador por excelencia, que incrementa de forma muy considerable el
caudal matemático griego, la trascendente tarea de Euclides estriba en estructurar el patrimonio
matemático griego en un entramado consistente, claramente organizado mediante una concatena-
ción lógica de los resultados – Los Elementos –.
A Euclides le cabe el inmenso mérito de la ordenación y sistematización de la Geometrı́a griega
elemental, de manera que con independencia de sus aportes originales, su mayor contribución se le
reconoce como gran compilador y creador de un estilo de exposición –el método axiomático–, de
modo que en lenguaje actual dirı́amos que Euclides es un gran maestro y su obra fundamental un
Libro de Texto, que establece un férreo paradigma de exposición y de demostración en Matemáticas,
una especie de norma académica de obligado respeto para todo matemático.
Las referencias más fiables sobre Euclides serı́an las que relata Proclo (siglo V d.C.) en su
Comentario al Libro I de Los Elementos de Euclides.
Aproximadamente 300 años antes de Cristo Euclides de Alejandrı́a escribió un tratado en trece
libros llamado Los Elementos. Proclo nos dice: “ Euclides, el autor de los Elementos ordenó diversos
trabajos de Eudoxo, mejoró los de Teeteto y produjo también demostraciones irrefutables para
aquello que sus predecesores no habı́an probado de manera rigurosa. Vivio en la época del primer
Tolomeo, quién una vez le preguntó si habı́a en geometrı́a un camino más corto que los elementos,
y él respondió que no hay un camino real a la geometrı́a.”
Los Elementos de Euclides constan de 465 Proposiciones organizadas en trece libros. De los
trece libros, los primeros seis se pueden describir como los que tratan con triángulos, rectángulos,
cı́rculos, polı́gonos, proporción y semejanza respectivamente. Los Libros VII, VIII y IX tratan de
Teorı́a de Números, es decir de las propiedades de los números enteros y la divisibilidad. El Libro X
introduce el Método de Exhaución y clasifica de forma sistemática los segmentos √ inconmensurables.
En estos últimos dos libros se prueba que hay infinitos números primos, y que 2 es irracional. Los
Libros XI y XII estudian la geometrı́a de sólidos aplicando el Método de Exhaución de Eudoxo al
cálculo del área del cı́rculo y algunos volúmenes. Finalmente el Libro XIII está dedicado al estudio
exhaustivo de cinco poliedros regulares.
Ası́ pues, aunque no hay ninguna introducción o preámbulo de la obra, previamente, en el Libro
I, Euclides introduce unos preliminares a base de veintitrés definiciones, cinco postulados y cinco
nociones comunes o axiomas.

Definición 1.1

1. Punto es lo que tiene posición pero no dimensiones.

2. Lı́nea es la longitud sin anchura.

3. Los extremos de la lı́nea son puntos.

4. Lı́nea recta es la que yace por igual sobre sus puntos.

5. Superficie es lo que sólo tiene largo y ancho.

6. Los extremos de la superficie son lı́neas.

7. Superficie plana es la que yace por igual sobre sus rectas.

UES-FCNM-Escuela de Matemática-Licenciatura en Matemática y Licenciatura en Estadı́stica


pág. 5
Geometrı́a I. 1 Aspectos históricos

8. Ángulo plano es la inclinación de dos lı́neas que se encuentran en un plano y no yacen las
dos sobre una recta.

9. Si las dos lı́neas que contienen el ángulo son rectas, el ángulo se llama rectilı́neo.

10. Si una recta trazada sobre otra forma con ella dos ángulos contiguos iguales cada uno de
ellos es recto, y la recta se llama perpendicular a aquella sobre la cual se trazó.

11. Ángulo obtuso es el mayor que el recto.

12. Ángulo agudo es el menor que el recto.

13. Lı́mite es el extremo de algo.

14. Figura es lo comprendido por uno o varios lı́mites.

15. Cı́rculo es una figura plana limitada por una sola lı́nea que se llama periferia [circunferencia],
respecto de la cual son iguales las rectas que inciden sobre ellas trazadas desde uno de los
puntos situados en el interior de la figura.

16. Este punto se llama centro del cı́rculo.

17. Diámetro del cı́rculo es una recta cualquiera que pase por el centro y cuyas dos partes tengan
sus extremos en la periferia. Esa recta divide al cı́rculo en dos partes iguales.

18. Semicı́rculo es la figura limitada por un diámetro y la periferia. El centro del semicı́rculo es
el mismo que el del cı́rculo.

19. Figuras rectilı́neas son las limitadas por rectas. Triláteras si lo están por tres, cuadriláteras
por cuatro y multiláteras por más de cuatro.

20. Entre las figuras triláteras el triángulo es equilátero si tiene los tres lados iguales, isósceles
si solo tiene dos lados iguales y escaleno si sus tres lados son desiguales.

21. Entre la figuras triláteras, el triángulo rectángulo es el que tiene un ángulo recto; obtusángulo,
el que tiene un ángulo obtuso, y acutángulo, el que tiene sus tres ángulos agudos.

22. Entre las figuras cuadriláteras, el cuadrado es equilátero y equiángulo; el rectángulo, equiángu-
lo, pero no equilátero; el rombo es equilátero, pero no rectangular; el romboide, sin ser
equilátero ni equiángulo, tiene iguales los lados y los ángulos opuestos. Las demás figuras
cuadriláteras se llaman trapecios.

23. Rectas paralelas son las que, estando en el mismo plano y prolongadas al indefinidamente,
no se encuentran.

Con base en estas definiciones, Euclides presenta a continuación una lista de cinco postulados
y cinco nociones comunes (o axiomas).

UES-FCNM-Escuela de Matemática-Licenciatura en Matemática y Licenciatura en Estadı́stica


pág. 6
Geometrı́a I. 1 Aspectos históricos

Postulados
1. (Es posible) trazar una lı́nea recta desde un punto cualquiera a otro punto cualquiera.
(Cuando se considera una lı́nea recta contenida entre dos puntos fijos, que son sus extremos,
tal porción se llama una lı́nea recta finita).

2. (Es posible) prolongar de una manera ilimitada en lı́nea recta una recta limitada.

3. (Es posible) describir un cı́rculo para cada centro y cada radio.

4. Todos los ángulos rectos son iguales.

5. Si una recta, al incidir sobre otras dos, forma del mismo lado ángulos internos menores que
dos rectos, las dos rectas prolongadas indefinidamente se encontrarán en el lado en que estén
los ángulos menores que dos rectos.

Nociones Comunes:(Axiomas)
Cosas iguales a una misma cosa son iguales entre sı́.

Si a cosas iguales se agregan cosas iguales, los totales son iguales.

Si de cosas iguales se quitan cosas iguales, los restos son iguales.

Las cosas que se superponen una a la otra son iguales entre sı́.

El todo es mayor que la parte.

La primera de estas Nociones Comunes es la ley transitiva que podemos considerar como el
silogismo fundamental de la Geometrı́a. Las dos siguientes se refieren a la legitimidad de sumar y
restar cosas iguales. La última introduce la desigualdad. La cuarta de las Nociones Comunes merece
una atención. Euclides viene a decir que si una figura se puede trasladar sobre el plano de modo
que al colocarse sobre otra, ambas figuras coinciden perfectamente –se superponen–, entonces las
dos figuras son iguales en todos sus aspectos, es decir, tiene los mismos ángulos, los mismos lados
y demás elementos.

UES-FCNM-Escuela de Matemática-Licenciatura en Matemática y Licenciatura en Estadı́stica


pág. 7
Geometrı́a I. 2 Segmentos

2. Segmentos
2.1. Introducción
Una figura geométrica es una combinación de puntos, lı́neas y planos. Los conceptos de punto,
lı́nea y plano son conceptos primitivos, que no se definen, únicamente pueden describirse. Por
ejemplo, un punto es un objeto que solo tiene posición. No tiene longitud, anchura ni espesor. Se
trata pués, de una idealización matemática.
La Geometrı́a es la ciencia que estudia la posición, forma y magnitud de las figuras geométricas.
La geometrı́a plana estudia las figuras cuyos puntos se encuentran en el mismo plano (triángulos,
cuadrados, cı́rculos, etc.). La geometrı́a sólida estudia las figuras que pueden estar en distintos
planos (pirámides, cubos, esferas, etc.).

2.2. Lı́neas
Una lı́nea es una figura geométrica que tiene longitud, pero no tiene anchura ni grosor.

Figura 2: Una linea recta

Se puede considerar que una lı́nea se genera con el movimiento de un punto.


Con este enfoque, una lı́nea recta se genera con el movimiento de un punto que se mueve siempre
en la misma dirección (ver Figura 2), mientras que una lı́nea curva se genera con el movimiento
de un punto que cambia continuamente de dirección (ver Figura 3).

Figura 3: Una linea curva

Otro punto de vista consiste en considerar una lı́nea como formada por infinitos puntos. En
particular, una lı́nea recta (llamada por brevedad recta) queda completamente determinada por
dos cualesquiera de sus puntos y se considera ilimitada en extensión.
Notación: Si A y B son dos puntos que pertenecen a una recta, dicha recta se denota como
←→
AB.

UES-FCNM-Escuela de Matemática-Licenciatura en Matemática y Licenciatura en Estadı́stica


pág. 8
Geometrı́a I. 2 Segmentos

2.3. Rayos
Dada una lı́nea recta, a una porción de dicha recta que se origina en un punto O y que se
extiende ilimitadamente por el extremo que contiene a un punto A, se le llama rayo OA (ver
−→
Figura 4) y se denota por OA.

Figura 4: rayo OA

2.4. Segmento de recta


Dados dos puntos distintos A y B en una recta, se le llama segmento a la figura formada por
A, B y todos los puntos que se encuentran entre ellos dos (ver Figura 5).

A B

Figura 5: Segmento AB

Se denota por AB (o simplemente AB) y se lee “segmento AB”. Los puntos A y B se llaman
extremos y los otros puntos forman el interior del segmento.
La medida de un segmento AB se denota m(AB) o simplemente AB y es un número positivo
que se compara con la longitud de un segmento unitario.

2.5. Punto medio de un segmento


Un punto C se llama punto medio de un segmento AB si C está entre A y B y se verifica que
AC = CB (ver Figura 6).

A C B

Figura 6: C punto medio del segmento AB

Todo segmento posee un punto medio el cual lo biseca, es decir, lo divide en dos segmentos de
igual longitud.

UES-FCNM-Escuela de Matemática-Licenciatura en Matemática y Licenciatura en Estadı́stica


pág. 9
Geometrı́a I. 2 Segmentos

x
A C B

a b

Figura 7:

2.6. Operaciones con segmentos colineales


Medir es comparar una magnitud con otra de su misma clase que sirve como patrón y a la que
se llama unidad de medida. Para medir los segmentos se utilizan diversos instrumentos, siendo el
más sencillo una regla graduada. En la Figura 7 se cumplen las siguientes relaciones:

AB = AC + CB ⇒ x=a+b (1)

AC = AB − CB ⇒ a=x−b

CB = AB − AC ⇒ b=x−a
Observación:
La relación de adición (1) se puede generalizar ası́: Tomemos “n” puntos consecutivos A1 , A2 , A3 , . . . , An
en una misma recta, entonces se verificará la siguiente relación:

A1 An = A1 A2 + A2 A3 + A3 A4 + · · · + An−1 An

Este resultado se conoce como el Teorema de Chasles. Debemos notar que los segmentos consi-
derados son consecutivos.

Ejemplo 2.1 Ejemplos de problemas con segmentos

a) Sobre una recta están ubicados los puntos A, B, C y D. Si AD = 24 cm, AC = 15 cm y


BD = 17 cm, ¿cuánto mide BC en cm?
Solución
Según los datos del problema (ver Figura 8), se tiene que:
CD = AD − AC = 24 − 15 = 9 cm
y puesto que: BC = BD − CD, se tiene que BC = 17 − 9 = 8 cm. 

b) Sean A, B, C y D puntos consecutivos de una recta. Si AC + BD = 16 m, y BC = 4 m,


¿cuál es el valor de AD en m?
Solución
Construimos la figura según las condiciones del problema y se observa que:
AD = AC + CD, pero CD = BD − 4.
z }| {
Luego, AD = AC + BD −4 = 16 − 4 = 12 m 

UES-FCNM-Escuela de Matemática-Licenciatura en Matemática y Licenciatura en Estadı́stica


pág. 10
Geometrı́a I. 2 Segmentos

A B C D

Figura 8:

c) M , A, O y B son puntos consecutivos sobre una recta, siendo O el punto medio de AB. Si
M A = 2 y AB = 6, calcular (M O)2 .
Solución
Ya que O es el punto medio de AB, se tiene:
AB 6
AO = OB = = =3
2 2
Luego, M O = M A + AO = 2 + 3 = 5
Por lo tanto, (M O)2 = 25 
d) Sobre una recta se tienen los puntos consecutivos A, B y D. Entre los puntos B y D se toma
un punto C tal que AC = CD/4. Determinar BC, sabiendo que BD − 4AB = 20.
Solución
CD
Sea x = BC. Del dato AC = 4
y llamando AC = a, se tiene CD = 4a.
Reemplazando los literales x y a en la igualdad BD − 4AB = 20, se tiene (dibujar la figura):
(x + 4a) − 4(a − x) = 20
x + 4a − 4a + 4x = 20
5x = 20
x=4
Es decir, BC = 4. 
e) Al dividir un segmento en partes cuyas medidas son directamente proporcionales a 1/3, 1/4
y 1/2 se obtienen tres segmentos, el segundo de los cuales mide 12 cm. ¿Cuál es la suma en
cm de las medidas del segundo y tercer segmento?
Solución
Sean los segmentos AB, BC y AC. Según las condiciones del problema:

k k k
AB = , BC = y CD =
3 4 2
k
ya que BC = 12 entonces 4
= 12, de donde k = 48.
48
Esto nos permite calcular el valor del tercer segmento CD, ası́ CD = 2
= 24.
Luego, la suma pedida es BC + CD = 36. 

UES-FCNM-Escuela de Matemática-Licenciatura en Matemática y Licenciatura en Estadı́stica


pág. 11
Geometrı́a I. 2 Segmentos

2.7. Ejercicios
1) Sobre una recta se consideran los puntos consecutivos A, B y C. Si AB = 8 cm y BC = 12 cm,
hallar AC.

2) Dados los puntos colineales y consecutivos A, B, C y D, tales que AB = 7, BC = 8 y


AD = 24, calcular CD.

3) En una lı́nea recta se consideran los puntos consecutivos A, B y C tales que AC = 25 y


BC = 15. Calcular AB.

4) A, B, C y D son puntos ubicados en una lı́nea recta de modo que AB = BC, CD = 20 y


AB = 5. Hallar AD.

5) Dado el segmento AB y su punto medio O, si P es un punto interior al segmento OB, OP = 1


y P B = 5, calcular AB.

6) En una lı́nea recta se ubican los puntos consecutivos A, B, C y D de modo que AB = 2BC
y CD = 3BC. Si BC = 1, calcular AD.

Definición 2.1 1. Razón: se llama razón, al cociente de dos cantidades, expresadas en la


a
misma magnitud, por ejemplo .
b
a c
2. Proporción: se llama proporción a la igualdad de dos razones. Por ejemplo = , a los
b d
términos a y d se les llama extremos y los términos b y c se les llama medios, al término d
se le llama cuarta proporcional entre a, b y c en este orden.
PA
3. Un punto P ∈ AB divide al segmento AB en una razón r si = r. Si r = 1 entonces P
PB
1
es el punto medio de AB.

A P B

Figura 9:

4. Sean AB y CD y sean X ∈ AB y Y ∈ CD, decimos que X e Y dividen a AB y CD en


segmentos proporcionales si
XA YC
=
XB YD

Ejemplo 2.2

A 8 P 2 B

PA 8
P divide AB en la razón = =4
PB 2
Consideremos las divisiones siguientes:
1
Cuando el puntos P está entre los puntos A y B decimos que P divide interiormente al segmento AB en la
razón r. Si P no está entre los puntos A y B, decimos que P divide exteriormente a segmento AB.

UES-FCNM-Escuela de Matemática-Licenciatura en Matemática y Licenciatura en Estadı́stica


pág. 12
Geometrı́a I. 2 Segmentos

A X B

C Y D

Figura 10:

PA x 1
A x P 2x B = =
PB 2x 2

N 3x A 3x B NA 3x 1
= =
NB 6x 2

Dado un segmento AB y una razón k 6= 1, conseguimos encontrar dos puntos que dividen AB
en esta razón: una interior y otra exterior. Cuando AB está dividido por dos punto P y N , en la
misma razón, decimos que el segmento AB está dividido armónicamente. Y, los puntos P y N se
llaman conjugados armónicos con respecto a A y B.

Definición 2.2 ( División armónica)


Decimos que los puntos P y N dividen armónicamente al segmento AB cuando
PA NA N A P B
= .
PB NB

Esta definición de división armónica es equivalente a los sigueinte: Se dice que dos puntos
dividen un segmento de lı́nea armónicamente si lo dividen interna y externamente en la misma
razón.

2.8. Ejercicios
1. Sobre una linea recta se consideran los puntos colineales y consecutivos A, B, C y D; tal que
AC = 19 y BD = 23. Encuentre la longitud del segmento que une los puntos medios de AB
y CD.

2. Sobre una linea recta se consideran los puntos colineales y consecutivos A, B, C y D; siendo
C el puto medio de BD; displaystyle CBCA
= 23 y AD = 12. Encuentre CD.

3. Sobre una lı́nea recta se consideran los puntos consecutivos M, N, P, Q tal que: P Q = 3N P
y 3M N + M Q = 4. Encuentre la longitud del segmento M P

4. Sobre una lı́nea recta se consideran los puntos consecutivos A, B y C y luego se ubican
los puntos medios M √y F de AB y M C respectivamente. Encuentre la longitud de AF . Si
AB + F C − AM = 2 5.

5. Sobre una lı́nea recta se consideran los puntos consecutvos A, B, C, D y luego se toman M y
F puntos medios de AB y CD respectivamente. Encuentre M F , si: AC = 18 y BD = 34.

6. Dado el segmento AB y un punto M interior a el. Demuestre que si el producto AM · M B,


es máximo entonces M es el punto medio de AB.

UES-FCNM-Escuela de Matemática-Licenciatura en Matemática y Licenciatura en Estadı́stica


pág. 13
Geometrı́a I. 2 Segmentos

7. A, B, C y D son puntos colineales y consecutivos tal que: AB ·CD = AD ·BC; AB ·.BC = x;


AD · CD = y. Calcule BD.

8. Sobre una ı́nea recta se consideran los puntos consecutivos A, B, C, D y D tal que: DC =
2 · AB; AB = a y BD = b. Encuentre AC.

9. A, B, C y D son puntos colineales


 y consecutivos.
 Si AC es la media proporcional2 entre AD
AD AB
y BD. Calcular k, si: k = 2 −1 .
AC CD

2
Dados dos segmentos de longitudes a y b se llama media proporcional a un segmento de longitud x, talque
a x
verifique = .
x b

UES-FCNM-Escuela de Matemática-Licenciatura en Matemática y Licenciatura en Estadı́stica


pág. 14
Geometrı́a I. 3 Ángulos

3. Ángulos
Definición 3.1 Definimos como ángulo a la figura geométrica formada por dos rayos (o semirrec-
tas) distintas que tienen el mismo origen. Ese origen se llama vértice del ángulo. Al ángulo de
vértice O y rayos OA y OB se le denota ∠AOB.

B
B
B

A
O

A A A
O O B
O

Ángulo agudo Ángulo recto


Ángulo obtuso

Figura 11: Ejemplo de ángulos

Dos ángulos ∠AOB y ∠BOC son adyacentes si y sólo si tienen un lado común OB y los lados
no comunes OA y OC están en semiplanos distintos, determinados por el lado común.

A
O

Figura 12: Ejemplo de angulos adyacentes

Bisectriz de un ángulo es la semirrecta que lo “divide” en dos ángulos adyacentes iguales.

La construcción con regla y compás de la bisectriz, se hace de la siguiente manera:


Hacemos centro en el vértice O del ángulo y abriendo el compás la medida que se quiera, se traza
un arco que corta a los lados del ángulo en los puntos C y D, Ahora tomamos una abertura del
compás, un poco mayor a la mitad del arco comprendido entre los puntos C y D. La abertura puede
ser cualquiera pero tiene que ser un poco mayor a la mitad del citado arco. Con esta abertura se
trazar un arco desde C y otro desde D. Se cortan en el punto E. Uniendo E con el vértice O del
ángulo, obtenemos la BISECTRIZ

Dos ángulos son:

UES-FCNM-Escuela de Matemática-Licenciatura en Matemática y Licenciatura en Estadı́stica


pág. 15
Geometrı́a I. 3 Ángulos

Figura 13:

Congruentes o Iguales: si tienen igual medida.


Suplementarios: si su suma es 180°.
Complementarios: si su suma es 90°.
Por otra parte, dos rectas en el plano pueden ser secantes o paralelas,3 dependiendo si se cortan
o no; además, si las rectas son secantes, el punto de corte es único, y definen cuatro ángulos, que
se agrupan por parejas en ángulos opuestos por el vértice (las parejas de ángulos tales que uno
está formado por la prolongación de los lados del otro).

C
B

D A

Los ángulos opuestos por el vértice son iguales (Justifique). Por ejemplo, ∠AOB = ∠COD.
Por lo que dos rectas secantes forman cuatro ángulos que definen dos parejas de ángulos iguales,
y si tomamos un miembro de cada pareja, se tienen dos ángulos suplementarios. En particular, si
las rectas son secantes y forman cuatro ángulos iguales, serán llamadas rectas perpendiculares,4 y
los ángulos ası́ generados son llamados ángulos rectos. Y como es muy conocido, un ángulo agudo
es aquel cuya medida es menor a la de un ángulo recto, y un ángulo obtuso es aquel cuya medida
es mayor que un ángulo recto; en particular, un ángulo obtuso será llamado ángulo llano si su
medida es el doble que la de un ángulo recto.

3.1. Ángulos formados por dos rectas y una trasversal a ellas.


Al intersecar un par de rectas por una recta llamada transversal o secante, se forman los
siguientes tipos de ángulo:
3
Si la recta AB es paralela a la recta CD, se denota AB k CD.
4
Si la recta AB es perpendicular a la recta CD, se denota AB ⊥ CD.

UES-FCNM-Escuela de Matemática-Licenciatura en Matemática y Licenciatura en Estadı́stica


pág. 16
Geometrı́a I. 3 Ángulos

β
α
γ θ
L1

ε φ
L2
ψ δ

Ángulos internos: γ, θ, ε, φ

Ángulos externos: α, β, ψ, δ

Ángulos Correspondientes: Son dos ángulos no adyacentes situados en el mismo lado de la


secante, uno en el interior y otro en el exterior: α y ε; β y φ; θ y δ; γ y ψ.

Ángulos Alternos Internos: Son dos ángulos no adyacentes situados en el interior, y en dis-
tintos lado de la secante: γ y φ; θ y ε.

Ángulos Alternos Externos: Son dos ángulos no adyacentes situados en el exterior, y en


distintos lado de la secante: α y δ; β y ψ.

Ángulos Conjugados: Son dos ángulos internos o externos, no adyacentes y situados del
mismo lado de la secante:

a) Ángulos conjugados internos: γ y ε; θ y φ.


b) Ángulos conjugados externos: α y ψ; β y δ.

Cuando las rectas L1 y L2 son paralelas (L1 k L2 ) se cumple que:


1. Los ángulos correspondientes son igua- L
les entre sı́.
β
2. Los ángulos alternos internos son igua- α
les entre sı́. L1 γ θ

3. Los ángulos alternos externos son igua-


les entre sı́. ε φ
L2
4. Los ángulos conjugados son suplemen- ψ δ
tarios.

UES-FCNM-Escuela de Matemática-Licenciatura en Matemática y Licenciatura en Estadı́stica


pág. 17
Geometrı́a I. 3 Ángulos

3.2. Problemas
1. Tres ángulos adyacentes forman un semiplano y tienen sus medidas proporcionales a los
números 5, 7 y 8. Hallar la medida del menor ángulo.

2. Demostrar que las bisectrices de dos ángulos suplementarios son perpendiculares.

3. En la figura adjunta, L1 k L2 y
L3 k L4 . Calcular x.

4. Con ayuda de la figura 14, demuestre que: Si L1 k L2 entonces γ = α + β.

Figura 14:

5. En la figura 15, AB k F G. Hallar el ángulo x si el ∠AM F = 90° y el ∠M AB = 110°.

Figura 15:

UES-FCNM-Escuela de Matemática-Licenciatura en Matemática y Licenciatura en Estadı́stica


pág. 18
Geometrı́a I. 3 Ángulos

6. Calcular el ∠OP Q, si OP es bisectriz del ángulo O, L1 k L2 y P Q ⊥ L1 . Ver figura 16.

Figura 16:

7. En la figura 17, L1 k L2 y L3 k L4 , calcular α.

Figura 17:

8. En la figura 18, calcular x, si L1 k L2 .

Figura 18:

UES-FCNM-Escuela de Matemática-Licenciatura en Matemática y Licenciatura en Estadı́stica


pág. 19
Geometrı́a I. 3 Ángulos

9. Calcular la medida θ del gráfico anexo, si las


rectas L1 y L2 son paralelas.

10. En la figura 19, L1 k L2 y L3 k L4 . Hallar el valor del ángulo θ.

Figura 19:

11. Sea ∠AOB = 24°, en la región exterior a dicho ángulo se traza el rayo OC. Hallar la medida
del ángulo formado por las bisectrices de los ángulos AOC y BOC.

12. Del gráfico 20, calcular y, cuando x tome su máximo valor entero.

x+y

2y − x x−y

Figura 20:

UES-FCNM-Escuela de Matemática-Licenciatura en Matemática y Licenciatura en Estadı́stica


pág. 20
Geometrı́a I. 4 triángulos

4. triángulos
Diremos que tres puntos que pertenecen a una misma recta son puntos colineales; de manera
análoga, si tres rectas pasan por un mismo punto, serán llamadas rectas concurrentes. Si tomamos
“al azar” tres puntos en el plano, en muy raras ocasiones estos puntos estarán alineados,5 y diremos
entonces que son los vértices de un triángulo; análogamente sucede con las rectas, tres rectas por
lo general no concurren, y la figura geométrica que éstas definen es también un triángulo.6 Una
definición completa para nuestros intereses es la siguiente:

Definición 4.1 (Definición de Triángulo)

Si A, B y C son tres puntos cualesquiera no colineales (Ver figura 21), entonces la reunión se los
segmentos AB, BC y AC se llama triángulo ABC y se denota por 4ABC. Los puntos A, B y
C se llaman vértices y los segmentos AB, BC y AC se llaman lados. Simbólicamente: 4ABC =
AB ∪ BC ∪ AC.

4.1. Clasificación de Triángulos


1. Con relación a sus lados:

a) Escaleno: si sus tres lados no son congruentes.


b) Isósceles: si por lo menos dos de sus lados son congruentes.
c) Equilátero: si sus tres lados son congruentes (note un triángulo equilátero es también
isósceles, y que los tres ángulos internos son iguales entre sı́ e iguales a 60)

2. Con relación a sus ángulos internos:

a) Acutángulo: si su ángulo mayor es agudo (note que entonces los tres ángulos son agudos)
b) Rectángulo: si su ángulo mayor es ángulo recto (note que el ángulo en cuestión es único
y que los otros dos ángulos son agudos; ası́, en un triángulo rectángulo, la hipotenusa
es mayor a los catetos)
c) Obtusángulo, si el ángulo mayor es ángulo obtuso (note que el ángulo en cuestión es
único y que los otros son agudos; ası́, en un triángulo obtusángulo, el lado que se opone
al ángulo obtuso es el lado mayor)

Todo triángulo ABC determina tres ángulos internos o interiores: ∠ABC, ∠ACB y ∠BAC,
y se llamará ángulo externo o exterior, al ángulo determinado por un lado y la prolongación del
lado adyacente, en la figura 21, α, β y θ son ángulos exteriores.
Dado el 4ABC, se tiene que AB + BC + CA = p = 2s, donde p es llamado el perı́metro y s el
semiperı́metro del triángulo. Para abreviar, suele asociarse a cada vértice un lado opuesto, y vice-
versa, por ejemplo, el lado opuesto de A es BC, y es frecuente que se denote por a; análogamente
b = CA, c = AB.

5
En teorı́a de probabilidades, ¡la probabilidad que esto ocurra es cero!
6
El término más riguroso para esta figura es trilátero. En este caso, habrı́a que hacer una consideración: si hay
un par de rectas paralelas, el trilátero definido ya no es “normal” según nuestro sentido común, sin embargo, ¡sigue
siendo un trilátero!

UES-FCNM-Escuela de Matemática-Licenciatura en Matemática y Licenciatura en Estadı́stica


pág. 21
Geometrı́a I. 4 triángulos

Figura 21: Elementos del Triángulo

4.2. Teoremas Fundamentales del Triángulo


(Cada teorema lo demostraremos en clase)

Teorema 1 En todo triángulo, la medida de un ángulo exterior es igual a la suma de las medidas
de dos ángulos interiores del triángulo no adyacentes a él.

La demostración de este teorema se basa en las relaciones de ángulos entre paralelas; se deja al lec-
tor que haga la demostración (Sugerencia: por un vértice, trace una recta paralela al lado opuesto)

Teorema 2 En todo triángulo, la suma de las medidas de sus tres ángulos internos es igual a
180°.

Teorema 3 (Desigualdad Triangular) En todo triángulo, la longitud de uno de sus lados está com-
prendido entre la suma y la diferencia de los otros dos.

Sin ser muy rigurosos, suponga que dado el segmento AB se traza con centro en A una circun-
ferencia de radio r1 , y con centro en B una circunferencia de radio r2 ; si AB < r1 + r2 , las circunfe-
rencias se cortarán en dos puntos, y cualquiera de ellos puede ser el vértice C, ası́ AB < BC + CA;
en cambio, si AB = r1 + r2 o peor aún, si AB > r1 + r2 , la construcción del 4ABC no es posible.

La Desigualdad Triangular es un resultado fundamental, a partir de ésta y de su modelo de de-


mostración se generan los Criterios de Congruencia de Triángulos; a groso modo, si dadas ciertas
condiciones, la construcción de una figura geométrica (un triángulo en particular) queda deter-
minada de manera única, entonces dos figuras que reunen las mismas condiciones serán llamadas
figuras congruentes.

Ası́, si se tienen tres segmentos (cuyas longitudes cumplen la desigualdad triangular), dejando
uno fijo y construyendo las circunferencias con centros en los extremos de este segmento y radios
las longitudes de los otros segmentos, por construcción, sólo será posible obtener dos triángulos
(uno con cada punto de intersección de las circunferencias), que son básicamente el mismo pero
la orientación de los ángulos es contraria; ası́, si se sabe que dos triángulos cumplen tener lados
respectivamente iguales, por construcción, deben de ser iguales. Este es el conocido criterio LLL

UES-FCNM-Escuela de Matemática-Licenciatura en Matemática y Licenciatura en Estadı́stica


pág. 22
Geometrı́a I. 4 triángulos

de congruencia de triángulos; más adelante se detallarán el resto de criterios, pero a partir de este
probaremos el siguiente resultado:

Teorema 4 En todo triángulo, se cumple que a lados iguales se oponen ángulos iguales, y vice-
versa.

Suponga que 4ABC es tal que AB = AC, entonces, por criterio LLL, 4ABC es congruente
al 4ACB (en ese orden, porque AB = AC, BC = CB y CA = BA), entonces, los ángulos que se
oponen a los ángulos iguales son iguales. Para el recı́proco necesitamos otro criterio de congruen-
cia, por lo que la demostración se dejará incompleta; retome esto en la sección de congruencia de
triángulos.

Teorema 5 En todo triángulo se cumple que a mayor lado se opone mayor ángulo y viceversa.

Este teorema se deja como ejercicio para el lector (Sugerencia: utilice el teorema anterior, tome
el lado mayor y defina un punto adecuado que genere un triángulo con dos lados iguales.)

4.3. Perpendicularidad
Mediatriz de un segmento
Se llama mediatriz del segmento AB a la recta que es perpendicular a este segmento y que pasa
por su punto medio. La mediatriz divide al segmento AB en otros dos segmentos de igual longitud.
La recta mediatriz tiene una importante propiedad: la distancia de cualquier punto de esa recta a
cada uno de los dos extremos del segmento AB es la misma.
Construcción de la mediatriz.
Vamos a construir la mediatriz de un segmento utilizando, como en casos anteriores, la regla y
el compás. Para ello representa dos puntos y traza el segmento que los une utilizando la regla.
Coloca el compás sobre uno de los extremos del segmento y ábrelo para que coincida con el otro
extremo. Traza ası́ una circunferencia. Haz la misma operación apoyando el compás sobre el otro
extremo. Une ahora los puntos donde se cortan las dos circunferencias que acabas de trazar. El nue-
vo segmento es perpendicular al inicial y si lo prolongas obtendrás la recta mediatriz que buscabas.

Figura 22:

UES-FCNM-Escuela de Matemática-Licenciatura en Matemática y Licenciatura en Estadı́stica


pág. 23
Geometrı́a I. 4 triángulos

Distancia de un punto a una recta. En la figura 23, sea P un punto exterior a una recta
L, la longitud de la perpendicular P M a la recta L es la distancia del punto P a dicha recta. Esta
perpendicular tiene la propiedad de ser única y su longitud es la distancia mı́nima del punto a la
recta (Pruébelo utilizano el hecho que la hipotenusa es mayor que los catetos). Los segmentos P A
y P B no son perpendiculares a L y se llaman oblicuas.

Figura 23:

Altura de un triángulo.
la altura es la menor distancia entre un vértice y el lado opuesto (o su prolongación), por lo que a
cada vértice le corresponde una altura. También utilizamos el nombre de altura para referirnos a
la recta que pasa por un vértice y es perpendicular al lado opuesto, pues es sobre esta recta sobre
la que medimos esa distancia.
Construcción de la altura.
Con C como centro y un radio suficientemente grande, construya un arco que corte a AB en P y Q.
Con P y Q como centros y un radio mayor que la mitad de P Q, construya arcos que se intersecten
en R. Trace CR que intersecta a AB en N. CN es la altura con respecto al lado AB.

Figura 24:

Teorema de Pitágoras
Abordamos el estudio de las Relaciones Métricas, del cual solo realizaremos el análisis del famoso
Teorema de Pitágoras, cuyo enunciado es el siguiente:

UES-FCNM-Escuela de Matemática-Licenciatura en Matemática y Licenciatura en Estadı́stica


pág. 24
Geometrı́a I. 4 triángulos

Teorema 6 (Teorema de Pitágoras) En un triángulo rectángulo, el cuadrado de la hipotenusa es


igual a la suma de los cuadrados de los catetos.

Una demostración de este teorema es debida a Thabit ibn Qurra (836-901), la cual consiste en
diseccionar la figura que se forma al construir dos cuadrados de lados respectivamente iguales a
los catetos de un triángulo rectángulo, como se muestra en el gráfico 25.

Figura 25:

Teorema 7 (Recı́proco del teorema de Pitágoras) Si en un triángulo el cuadrado de un lado es


igual a la suma de los cuadrados de los otros dos lados, el triángulo es rectángulo.7 .

4.4. Rectas Notables de un triángulo


1. Altura: Se llama altura de un triángulo al segmento que parte de uno de sus vértices y llega
en forma perpendicular al lado opuesto o a su prolongación.

2. Mediana: Se llama Mediana al segmento que une un vértice con el punto medio del lado
opuesto.

3. Mediatriz: Se denomina mediatriz de un lado de un triángulo es la recta perpendicular a


dicho lado en su punto medio.

4. Una Bisectriz: La bisectriz es la recta que “divide” en dos ángulos iguales a un ángulo dado;
en particular, es bisectriz interna si es la bisectriz de un ángulo interno de un triángulo, y
bisectriz externa si es la bisectriz de un ángulo externo de un triángulo.

4.5. Ejercicios

1. En la figura adjunta ambos triángulos son


equiláteros. Encuentre el valor de ϕ.

7
Ver demostración en la sección de congruencia de triángulos (página 32)

UES-FCNM-Escuela de Matemática-Licenciatura en Matemática y Licenciatura en Estadı́stica


pág. 25
Geometrı́a I. 4 triángulos

2. En la figura 26, calcular el ∠x si el ∠AOB = 100° y L1 k L2 .

Figura 26:

3. (*) En la figura 27, ABDE es un cuadrado y BCD es un triángulo isósceles con BD = DC.
Si ∠ABC = 160°, determinar la medida de ∠AEC.

Figura 27:

4. (*) (XV Competencia de Clubes Cabri Primera Ronda) En la figura


adjunta, ABCD es un rectángulo tal que AB = 2BC. M es el punto
medio de AB y los triángulos AM E y M BF son equiláteros. Si P
es la intersección de las rectas DE y CF , encuentre los ángulos del
4CDP .

5. Probar que una bisectriz exterior de un triángulo es paralela al lado opuesto si y sólo si el
triángulo es isósceles.
θ
6. Si AB y F G son rectas paralelas, el ∠ABC = ∠CDE = θ, el ∠DEF = 2
y el ∠GF H = 150°.
Calcule θ. Figura 28

UES-FCNM-Escuela de Matemática-Licenciatura en Matemática y Licenciatura en Estadı́stica


pág. 26
Geometrı́a I. 4 triángulos

Figura 28:

7. (*) Hallar la suma de los ángulos α +  + θ + φ en la figura 29.

Figura 29:

8. Determine el valor de la suma ∠A + ∠B + ∠I + ∠H + ∠F + ∠G. Figura 30.

Figura 30:

9. En el 4ABC el ∠BAC = 36° y AC = AB. Probar que la bisectriz interior BD (D en AC)


es congruente con el lado BC.

UES-FCNM-Escuela de Matemática-Licenciatura en Matemática y Licenciatura en Estadı́stica


pág. 27
Geometrı́a I. 4 triángulos

10. Sea ABC un triángulo rectángulo en B con AB = BC, se construye exteriormente el triángu-
lo equilátero BCD. Encuentre el ángulo ∠DAB.

11. En el 4ABC, AB = AC y D un punto sobre la recta AC, tal que BC = BD = DA.


Determine la medida del ángulo ∠ABD, si:

a) D está entre A y C.
b) A está entre D y C.

12. En un 4ABC, D es un punto sobre el lado AC tal que AB = AD. Si ∠ABC −∠ACB = 90°,
hallar el ∠CBD.

13. Se tiene un triángulo isósceles ABC, AB = BC en el cual se traza al altura AF tal que
BF = 6 y F C = 2. Hallar AC.

14. En la figura 31, el ∠ABC = ∠ACE, DC = EC, ¿Qué lı́nea notable es AD del 4BCA?

Figura 31:

15. ¿Cuál es el valor de b − a en la figura 32?

Figura 32:

16. (*) Sea ABC un triángulo tal que las medianas respectivas a B y C son perpendiculares.
Demuestre que se cumple la relación.

5BC 2 = CA2 + AB 2 .

UES-FCNM-Escuela de Matemática-Licenciatura en Matemática y Licenciatura en Estadı́stica


pág. 28
Geometrı́a I. 4 triángulos

17. La hipotenusa BC de un triángulo rectángulo ABC se divide en 4 segmentos congruentes


por los puntos G, E y H. Si BC = 20, encuentra la suma de los cuadrados de las longitudes
de los segmentos AG, AE y AH. Figura 33.

Figura 33:

18. (*) Dado un cuadrado ABCD, se construyen los triángulos equiláteros ABP (exteriormente)
y ADQ (interiormente). Probar que C, P y Q están alineados.

19. (*) Sea ABC un triángulo rectángulo con ∠CAB = 90°. D es un punto sobre la prolongación
de BC tal que BD = BA. E es un punto en el mismo semiplano que A respecto de BC, tal
que CE ⊥ BC y además CE = CA. Mostrar que A, D y E están alineados.

20. El cuadrilátero ABCD mostrado en la figura 34 cumple que AB k CD y BC k DA.8 Sobre las
prolongaciones de AB y AD se construyen puntos E y F tales que BC = BE y DC = DF .
Demuestre que C, E y F están alinedos.

Figura 34:

21. (*) En la figura adjunta, AB = BC = CD =


DE = EF = F G = GA. Calcule la medida del
∠DAE.

8
El cuadrilátero ABCD es un paralelogramo.

UES-FCNM-Escuela de Matemática-Licenciatura en Matemática y Licenciatura en Estadı́stica


pág. 29
Geometrı́a I. 4 triángulos

22. (*) (XXVIII Olimpiada Brasileña de Matemática) En la figura 35, AB = AC, AM = AN y


∠CAM = 30°, encuentre el valor del ∠BM N .

Figura 35:

23. Los lados de un triángulo isósceles son 12 y 5 metros, ¿cuál es su perı́metro?


a+b+c
24. Muestre que los lados de un triángulo cumplen que |a − b| < c y que c < 2
.

25. Muestre que es posible construir un triángulo con segmentos de longitudes a, b, c si y sólo
existen números positivos x, y, z tales que: a = x + y, b = y + z, c = z + x.

26. (*) (Etapa semifinal Estatal de XXII Olimpiada Mexicana de Matemáticas) En la figura
36 se muestra un hexágono regular ABCDEF de lado 1. Los arcos del cı́rculo que están
dibujados tienen centro en cada vértice del hexágono y radio igual a la distancia al vértice
opuesto. P , Q, R, S, T y U son los puntos de corte de estos arcos. ¿Cuánto mide cada lado
del hexágono P QRST U ?

Figura 36:

4.6. Congruencia de Triángulos.


Definición 4.2

El 4ABC es congruente al 4A0 B 0 C 0 si: AB = A0 B 0 , AC = A0 C 0 , BC = B 0 C 0 , ∠ABC = ∠A0 B 0 C 0 ,


∠ACB = ∠A0 C 0 B 0 y ∠BAC = ∠B 0 A0 C 0 . Simbólicamente: 4ABC = 4A0 B 0 C 0 . Véase figura 37.

UES-FCNM-Escuela de Matemática-Licenciatura en Matemática y Licenciatura en Estadı́stica


pág. 30
Geometrı́a I. 4 triángulos

Figura 37: Definición de Igualdad de Triángulos.

La definición anterior establece que dos triángulos son congruentes si tanto los lados como los
ángulos se presentan en pares respectivos congruentes. Esto, según la visión de Euclides, significa
que un triángulo es posible superponerlo sobre el otro (se puede desplazar, girar o reflejar) y coin-
cidirá de manera perfecta. Sin embargo, es importante mencionar que en muy raras ocasiones se
tendrá a disposición tanta información, de allı́ la importancia de los criterios de congruencia, que
establecen los requisitos mı́nimos para garantizar que dos triángulos son congruentes.

4.6.1. Criterios de Congruencia de triángulos


El siguiente es el primero de los tres criterios de congruencia de triángulos, y se denomina
criterio de LADO-ÁNGULO-LADO, en sı́mbolos: L-A-L.

Criterio L-A-L. Si los triángulos ABC y A0 B 0 C 0 presentan las congruencias: AB = A0 B 0 ,


AC = A0 C 0 y ∠BAC = ∠B 0 A0 C 0 , entonces 4ABC = 4A0 B 0 C 0 .

Figura 38: Criterio LAL

Según el criterio L-A-L, dos triángulos son congruentes si en uno de ellos existen dos lados y
el ángulo (comprendido entre dichos lados), respectivamente congruentes a dos lados y el ángulo
(comprendido entre dichos lados), en el otro triángulo.

Criterio A-L-A. Sean ABC y A0 B 0 C 0 dos triángulos tales que: AC = A0 C 0 , ∠BCA = ∠B 0 C 0 A0


y ∠BAC = ∠B 0 A0 C 0 , entonces 4ABC = 4A0 B 0 C 0 .

UES-FCNM-Escuela de Matemática-Licenciatura en Matemática y Licenciatura en Estadı́stica


pág. 31
Geometrı́a I. 4 triángulos

Figura 39: Criterio ALA.

Criterio L-L-L. Si un triángulo tiene sus tres lados respectivamente congruentes a los tres
lados de otro triángulo, entonces estos dos triángulos son congruentes.

Figura 40: Criterio LLL.

Ahora demostraremos el Recı́proco del Teorema de Pitágoras(página 25).

Demostración: Sea ABC un triángulo talque BC 2 = AB 2 + AC 2 , por construcción sea el


4A0 B 0 C 0 rectángulo en A0 tal que A0 B 0 = AB y A0 C 0 = AC, entonces por el teorema de Pitágoras
B 0 C 02 = A0 B 02 + A0 C 02 , ası́ que B 0 C 02 = BC 2 , de donde B 0 C 0 = BC y por el criterio LLL, se deduce
que el 4A0 B 0 C 0 = 4ABC, por lo tanto el ∠BAC = ∠B 0 A0 C 0 = 90°.

4.6.2. Teorema de la Base Media


En todo triángulo, el segmento que une los puntos medios de dos lados es paralelo al tercer
lado e igual a su mitad.

Figura 41: Teorema de La Base Media.

En la figura 41, M N es el segmento que une los puntos medios de los lados AB y BC del 4ABC,
AC
a este segmento se le llama BASE MEDIA DEL TRIÁNGULO. Se verifica que M N = y que
2

UES-FCNM-Escuela de Matemática-Licenciatura en Matemática y Licenciatura en Estadı́stica


pág. 32
Geometrı́a I. 4 triángulos

M N k AC.
Demostración:
1. Prolongar el segmento M N hasta el punto P tal que M N = N P .

2. Los triángulos M N B y P N C son congruentes, ya que BN = N C, M N = N P y el ∠CN P =


∠M N B, por consiguiente, el ∠N CP = ∠M BN , por lo tanto, CP k M B (Por ángulos
alternos internos iguales). Además, P C = M B = M A; con lo cual se tiene que: M A = P C.

3. Uniendo el punto A con el punto P se forman los triángulos congruentes AM P y ACP (por
L A L) ya que M A = P C, AP = AP , ∠M AP = ∠AP C (por ángulos alternos internos
entre las paralelas M A y P C). Luego, M P = AC, entonces N P = 12 M P = 12 AC. Además,
∠P AC = ∠M P A, de donde M P k AC o que M N k AC.
Corolario: Menor mediana de un triángulo rectángulo. En todo triángulo rectángulo,
la mediana relativa a la hipotenusa es la mitad de la longitud de la hipotenusa y es la menor de
las tres medianas del triángulo.

Demostración: En la figura 42, BM es la mediana relativa a la hipotenusa AC del 4ABC,


probaremos que BM = AC 2
; (con lo cual se tendrá que BM = AM = M C). Si por M se traza
una paralela al lado AB, que corte al lado BC en N , entonces N es el punto medio de BC y
el ∠M N C = 90°, los triángulos BN M y CN M son congruentes por el criterio L-A-L, luego
M B = M C = AM .
Probar que BM es la menor mediana (Ejercicio).

Figura 42: Menor Media en un Triángulo Rectángulo.

4.6.3. Ejercicios

1. (*) En la figura adjunta, ABC es un triángulo equilátero


y CDEF es un cuadrado. Se construye un punto G tal
que CF = CG y además ∠CF G = 15°. Probar que
∠AGC = ∠BDC.

2. Dado un triángulo equilátero ABC, se construye un triángulo equilátero DEF cuyos vértices
están sobre los lados del 4ABC, tal como muestra la figura 43. Demuestre que los triángulos
ADF , BED, CF E son todos congruentes entre si.

UES-FCNM-Escuela de Matemática-Licenciatura en Matemática y Licenciatura en Estadı́stica


pág. 33
Geometrı́a I. 4 triángulos

Figura 43:

3. ABCD es un cuadrado, E, F , G y H son puntos sobre los lados AB, BC, CD, DA, respec-
tivamente, tal que EF GH también es cuadrado. Demuestre que los triángulos AEH, BF E,
CGF , DHG son todos congruentes entre si. Figura 44.

Figura 44:

4. ABCDE y F GHIJ son pentágonos regulares (Vease figura 45). Demuestre que los triángulos
AF J, BGF , CHG, DIH, EJI son todos congruentes entre si.

Figura 45:

UES-FCNM-Escuela de Matemática-Licenciatura en Matemática y Licenciatura en Estadı́stica


pág. 34
Geometrı́a I. 4 triángulos

5. Si AB k CD y AB = CD entonces, AD = BC y AD k BC 9 .

6. Demuestre que dos triángulos desplazados


son congruentes. Sugencia: Utilice el pro-
blema anterior.

7. Demuestre que dos triángulos rotados son congruen-


tes.

10
8. Demuestre que dos triángulos reflejados con respecto a un punto son congruentes.

9. Demuestre que dos triángulos reflejados con respecto a


una recta son congruentes.

9
El cuadrilátero ABCD se denomina paralelogramo.
10
La reflexión con respecto a un punto es equivalente a una rotación de 180°

UES-FCNM-Escuela de Matemática-Licenciatura en Matemática y Licenciatura en Estadı́stica


pág. 35
Geometrı́a I. 4 triángulos

Importante: Las traslaciones, rotaciones y reflexiones no cambian el tamaño ni la forma de


un triángulo.

10. (*) En la figura adjunta, ABCD un cuadrado y EF ⊥ GH.


Demuestre que que EF = GH.

11. Dos cuadrados ABCD y EHGF , ambos de lado l,


están colocados en manera tal que un vértice de uno
está en el centro del otro (como en la figura anexa).
l2
Demuestre que el área del cuadrilátero EJBK es
4
y por ende no depende de la posición de J (o K).

12. En un 4ABC el ∠B = 2∠C, la mediatriz del lado AC corta en F al lado BC. Hallar AB,
si F C = 9.

13. En la figura 46, AC = 12 AF = 4 y ∠BAF = 30. Hallar BF si AG = GC.

Figura 46:

14. En la figura 47, AG = GC, el ∠AF G = 20°. Hallar el ∠F AC, si AC = 2BF .

UES-FCNM-Escuela de Matemática-Licenciatura en Matemática y Licenciatura en Estadı́stica


pág. 36
Geometrı́a I. 4 triángulos

Figura 47:

15. (*) (Examen final de XVI Olimpiada Mexicana de Matemática) Los ángulos de un triángulo
ABC están en progresión aritmética (∠B − ∠A = ∠C − ∠B = θ), D, E, y F son los puntos
medios de los lados BC, CA y AB, respectivamente. Llamamos H al pie de la altura trazada
desde C (que cae entre B y F ) y G a la intersección entre DH y EF . Hallar los ángulos del
4F GH.

16. (*) Sea ABCD un cuadrado. Se construyen triángulos equiláteros ADP y ABQ como se
muestra en la figura 48. Sea M la intersección de CQ con AD y N la intersección de CP
con AB. Demuestre que CM N es un triángulo equilátero.

Figura 48:

17. En la figura 49, ABC, CDE y EF A son triángulos isósceles, con el ∠ABC = ∠CDE =
∠EF A = 120°. Probar que el 4BDF es equilátero.

UES-FCNM-Escuela de Matemática-Licenciatura en Matemática y Licenciatura en Estadı́stica


pág. 37
Geometrı́a I. 4 triángulos

Figura 49:

18. (*) 4ABC es un triángulo isósceles con ∠ABC = ∠ACB = 80°. D es un punto en AC tal
que ∠ABD = 10°. Demuestre que AD = BC.

UES-FCNM-Escuela de Matemática-Licenciatura en Matemática y Licenciatura en Estadı́stica


pág. 38
Geometrı́a I. 5 Cuadriláteros

5. Cuadriláteros
5.1. Clasificación de Cuadriláteros (De acuerdo a sus diagonales
Los cuadriláteros pueden clasificarse de acuerdo a sus diagonales de la siguiente forma:

Cuadrilátero Convexo: Es un cuadrilátero con las dos diagonales en su interior.


A

B
C

Cuadrilátero Entrante: Es un cuadrilátero con una diagonal en el interior y otra en el exte-


rior.
A

B
C

Cuadrilátero Cruzado Es un cuadrilátero con las diagonales en su exterior.


B
A

D
11

Es muy frecuente que se considere que un cuadrilátero es convexo, a menos que se especifique
lo contrario. Esto es ası́ porque muchos resultados son más claros en un cuadrilátero convexo, sin
embargo, es importante darse cuenta que existen teoremas que no se cumplen para cualquier tipo
de cuadriláteros, por ejemplo:

Teorema: La suma de los ángulos internos de un cuadrilátero no cruzado es 360◦ .

La demostración de este resultado se basa en la disección del cuadrilátero en dos triángulos


cuyos ángulos internos conforman los ángulos internos del cuadrilátero, sin embargo, estas condi-
ciones no pueden lograrse en un cuadrilátero cruzado; de hecho, la suma de los ángulos internos
11
Tanto los cuadriláteros convexos como los entrantes son cuadriláteros simples, que son los cuadriláteros cuyos
lados no se cortan salvo en los extrenos; en contraposición, los cuadriláteros cruzados no son simples.

UES-FCNM-Escuela de Matemática-Licenciatura en Matemática y Licenciatura en Estadı́stica


pág. 39
Geometrı́a I. 5 Cuadriláteros

puede hacerse arbitrariamente pequeña cuando el cuadrilátero es cruzado.

También hay otras clasificaciones de cuadriláteros de acuerdo a sus lados y ángulos.

Cuadrilátero Equiángulo: un cuadrilátero (convexo) es equiángulo si todos sus ángulos in-


ternos son iguales; dado el teorema anterior, los ángulos son iguales a 90◦ , por ello este cuadrilátero
es llamado rectángulo.

Cuadrilátero Equilátero: un cuadrilátero (convexo) es equilátero si todos sus lados son igua-
les. A este cuadirátero también se le conoce como rombo.

Cuadrado: es un cuadrilátero que es equiángulo y equilátero.

C B

D A

Paralelogramo: es un cuadrilátero con los lados opuestos paralelos.

Trapecio: es un cuadrilátero con un par de lados opuestos paralelos.12

5.2. Paralelogramos
Es el cuadrilátero que tiene sus lados opuestos paralelos y congruentes. En todo paralelogramo
se cumple que sus ángulos opuestos son congruentes y sus diagonales se bisecan. El paralelogramo
también se conoce como romboide.
D
C

A
F B

Dado el paralogramo ABCD, por propiedades de ángulos entre paralelas es posible probar el
siguiente resultado:

Teorema: Los ángulos opuestos son iguales y los ángulos consecutivos son suplementarios:
∠ABC = ∠CDA = θ y ∠BCD = ∠DAB = 180 − θ.

12
Note que un paralelogramo es también un trapecio.

UES-FCNM-Escuela de Matemática-Licenciatura en Matemática y Licenciatura en Estadı́stica


pág. 40
Geometrı́a I. 5 Cuadriláteros

Por otra parte, por criterio ALA, 4ABC ≡ 4CDA; esto implica que AB = CD y BC = DA,
i.e.

Teorema: Los lados opuestos de un paralogramos son iguales.

A partir de esto, si M es la intersección de AC con BD, por criterio ALA, 4ABM ≡ 4CDM ,
por lo que AM = CM y BM = DM , i.e.

Teorema: Las diagonales de un paralelogramo se bisecan.

Además, se cumple un resultado sofisticado y muy importante:

Teorema: Ley del Paralelogramo. Si ABCD es un paralelogramo entonces el doble de la


suma de los cuadrados de los lados es igual a la suma de los cuadrados de las diagonales, es decir
2 AB 2 + BC 2 = AC 2 + BD2


.
Demostración: Aplicando la Ley del Coseno a 4ABC y 4ABD se tiene
AC 2 = AB 2 + BC 2 − AB · BC cos θ
DB 2 = AB 2 + AD2 − AB · AD cos(180 − θ)
⇒ AC 2 + DB 2 = 2 AB 2 + BC 2 − AB · BC (cos θ + cos(180 − θ))


y dado que cos θ = − cos(180 − θ) el resultado se sigue inmediatamente.

5.3. Rectángulos
Rectángulo: Sus cuatro ángulos son igual a 90°, sus lados opuestos son iguales y paralelos.
D C

A B

En primer lugar, es importante notar que todo rectángulo es paralelogramo (por ángulos entre
paralelas), por lo que todos los resultados probados anteriormente son heredados a todo rectángulo;
pero los rectángulos tienen propiedades adicionales:

Observe que por criterio LAL, 4ABC ≡ 4ABD, por lo que AC = BD y entonces
Teorema: Las diagonales de un paralelogramo son iguales; además, el punto de intersección
de estas equidista de los cuatro vértices y por tanto es el centro de una circunferencia que pasa
por todos los vértices.

Por otra parte, observe que si se aplica la ley del paralelogramo a un rectángulo se obtiene el
Teorema de Pitágoras.

UES-FCNM-Escuela de Matemática-Licenciatura en Matemática y Licenciatura en Estadı́stica


pág. 41
Geometrı́a I. 5 Cuadriláteros

5.4. Rombos
Rombo: Sus cuatro lados son iguales.
B

A
C

Dado un rombo ABCD, por criterio LLL, 4ABC ≡ 4CDA, y por lo tanto ∠BAC = ∠DAC
y ∠BCA = ∠DAC, lo cual implica BC k AD y AB k CD, i.e., todo rombo ABCD es un parale-
logramo. Además, por las mismas congruencias se tiene

Teorema: Las diagonales de un rombo cumplen ser una mediatriz de la otra.

Teorema: Las diagonales de un rombo bisecan a los ángulos interiores del rombo; esto implica
que el punto de corte de las diagonales equidista de los cuatro lados del rombo y es el centro de
una circunferencia tangente a estos.

Observaciones.
1. Un cuadrilátero es un para lelogramo si se cumple cualquiera de las siguientes condiciones:

a) Sus lados opuestos son paralelos.


b) Sus lados opuestos son congruentes.
c) Dos lados son congruentes y paralelos.
d ) Sus ángulos opuestos son congruentes.
e) Sus diagonales se bisecan entre sı́.

2. Se cumple la siguiente relación de inclusı́on.

Paralelogramo

Rectángulos Cuadrados
Rombos

UES-FCNM-Escuela de Matemática-Licenciatura en Matemática y Licenciatura en Estadı́stica


pág. 42
Geometrı́a I. 5 Cuadriláteros

5.5. Trapezoides
Es un cuadrilátero que no tiene pares de lados paralelos. Los trapezoides se clasifican en:
Trapezoide asimétrico: No tiene ningún par de lados paralelos o congruentes.
A

B
C

Trapezoide simétrico: Dos pares de lados consecutivos son congruentes; además una de las
diagonales es mediatriz de la otra.
B

A C

5.6. Trapecios
Es el cuadrilátero que tiene dos lados paralelos denominados base y los otros dos no son para-
lelos. La distancia entres sus bases se llama altura, y el segmento que une los puntos medios de los
lados no paralelos se denomina mediana.
Sea M y N los puntos medios de AD y BC. AB k DC.

D Base MenorC

E F

Base Mayor B
A G

Los trapecios se clasifican en: Trapecio escaleno: Es aquel en que sus lados no paralelos son
diferentes.

UES-FCNM-Escuela de Matemática-Licenciatura en Matemática y Licenciatura en Estadı́stica


pág. 43
Geometrı́a I. 5 Cuadriláteros

D C

B
A

Trapecio isósceles:Es aquel en sus lados no paralelos son congruentes. AB = CD

D C

B
A

Trapecio rectángulo: Cuando uno de sus lados no paralelos es perpendicular a las bases.

D C

B
A

Dado el trapecio ABCD (con AB k CD), se construyen los puntos medios de BC y DA, M
y N , respectivamente. Si el cuadrilátero M N AB se rota con centro en M y ángulo 180 se genera
un cuadrilátero M N 0 A0 C; observe que N D = N 0 A0 y N D k N 0 A0 , por lo que DN N 0 A0 es un
paralelogramo y

N N 0 = DA0
2M N = DC + CA0
2M N = DC + AB
AB + CD
⇒ MN =
2
El segmento M N es llamado base media del trapecio, y por lo recién demostrado se tiene

Teorema: La base media de un trapecio es igual a la semisuma de las bases.

Por otra parte, hay ciertos trapecios que reciben nombres particulares; el trapecio rectángulo
es aquel que las bases son perpendiculares a alguno de los otros lados; y por otra parte, el trapecio
isósceles es aquel que los lados (distintos de las bases) tienen igual longitud. 13
13
Los trapecios isósceles son muy importantes cuando se estudian los ángulos en la circunferencia; resulta que un
trapecio es isósceles si y sólo si los cuatro vértices se ubican sobre una misma circunferencia.

UES-FCNM-Escuela de Matemática-Licenciatura en Matemática y Licenciatura en Estadı́stica


pág. 44
Geometrı́a I. 5 Cuadriláteros

Área de un cuadrilátero: ¿ Qué es el área de una figura plana ?


Vamos a llamar área a la medida de la superficie que hay dentro de una figura en dos dimensiones,
es decir, una figura plana. Por ejemplo, observemos las siguientes figuras:

Figura 50:

Para calcular el área de una superficie debemos compararla con otra que elegimos como unidad
de superficie, y averiguar el número de unidades que contiene. Teniendo en cuenta la definición que
hemos visto para el área de una figura, podemos aplicarla a figuras sencillas y obtener expresiones
generales para cada una de ellas. Consideremos el siguiente rectángulo de 8cm de ancho y 4cm
de altura. Realizamos un análisis similar al del cuadrado. Si tomamos como unidad de medida la
superficie que ocupa un cuadrado que tiene lados de longitud 1, podemos observar que el cuadrado
pequeño cabe 32 veces en el rectángulo. Entonces el área del rectángulo es igual a 32.

5.7. Problemas
1. Dado el trapecio ABCD con AB k CD, demuestre que la bisectriz interior del ∠A es paralela
a la bisectriz exterior del ∠D.

2. A un rombo ABCD se le construyen exteriormente los cuadrados ABEF y BCGH. De-


muestre que 4ABD = 4EBH.

3. (*) Sea ABCD un paralelogramo. Se construyen triángulos equiláteros exteriores 4CDP y


4ADQ. Demuestre que el 4BP Q es equilátero.

4. Demuestre que las bisectrices interiores de un paralelogramo forman un rectángulo (¿qué su-
cede si el paralelogramo es además rombo?).

5. Demuestre que las bisectrices exteriores de un paralelogramo forman un rectángulo.

6. Sea ABCD un paralelogramo. La bisectriz interna del ∠CDA corta a BA en M , y la bisectriz


interna del ∠BAD corta a CD en N . Demuestre que ADN M es un rombo.

7. Demuestre que si por el punto de intersección de las diagonales de un rombo se trazan perpen-
diculares a los lados del rombo, entonces los puntos de intersección de dichas perpendiculares
con los lados del rombo forman un rectángulo.

8. Demuestre que las bisectrices de los ángulos definidos por las diagonales de un rombo, cortan
a los lados del rombo en cuatro puntos que forman un cuadrado.

9. En un 4ABC sea G la intersección de las medianas BB 0 y CC 0 . Sean B 00 , C 00 las reflexiones


de G respectivas a los puntos B 0 y C 0 .

UES-FCNM-Escuela de Matemática-Licenciatura en Matemática y Licenciatura en Estadı́stica


pág. 45
Geometrı́a I. 5 Cuadriláteros

a) Demuestre que AGCB 00 y AGBC 00 son paralelogramos.


b) A partir de lo anterior, demuestre que BCB 00 C 00 también es paralelogramo.
c) Demuestre que A0 pertenece a la recta AG, y concluya que las tres medianas de un
triángulo concurren en el punto G, llamado el centroide del 4ABC.
d) Demuestre que CG = 2GC 0 ; relaciones similares se cumplen para las otras dos medianas.

10. Teorema de Varignon: Dado un cuadrilátero ABCD (no necesariamente convexo), se


construyen los puntos medios L, M , N , O, P , Q, de los segmentos de recta AB, BC, CD,
DA, BD, AC, respectivamente. Figura 51.

a) Demuestre que LM N O, LP N Q, OP M Q, son paralelogramos.


b) Demuestre que LN , OM , P Q concurren en un punto, llamado el centroide del cuadrilátero
ABCD.
c) Demuestre que el perı́metro de LM N O es igual a AC + BD; resultados similares se
cumplen para los otros paralelogramos.

Figura 51: Teorema de Varignon

11. Sea ABCD un paralelogramo tal que existe un punto E sobre el lado AB que cumple
∠CED = 90. Sean M y N los pies de las perpendiculares trazadas desde A y B hacia DE
y CE, respectivamente. Demuestre que AC, BD y M N concurren.

12. (*) (Héctor Alberti) Sea ABCD un cuadrado. Se construyen los triángulos equiláteros BDA0 ,
ACB 0 , BDC 0 y ACD0 . Demuestre que el A0 B 0 C 0 D0 es también un cuadrado.

13. (*) (II Olimpiada Matemática del Cono Sur) En la figura 52 ABCD y AECF son paralelo-
gramos. Demuestre que BEDF es paralelogramo.

14. (*) ABCD es un cuadrilátero convexo y O es un punto en su interior. Sean P , Q, R, S, los


puntos medios de los lados AB, BC, CD, DA, respectivamente. Por P se traza una paralela
a OR, por Q se traza una paralela a OS, por R se traza una paralela a OP , y por S se traza
una paralela a OQ. Demuestre que estas cuatro rectas concurren.

15. (*) Un trapecio isósceles tiene diagonales perpendiculares y su área es 2010, determine su
altura.

UES-FCNM-Escuela de Matemática-Licenciatura en Matemática y Licenciatura en Estadı́stica


pág. 46
Geometrı́a I. 5 Cuadriláteros

Figura 52:

16. (*) (IX Competencia de Clubes Cabri, Segunda Ronda) Sea ABCDEF un hexágono regular
cuyo centro es O. Se construyen los cuadrados F SOP y ORCQ. Demuestre que AP QB y
SEDR son rectángulos. Figura 53.

Figura 53:

17. (*) Sobre los lados del 4ABC se trazan exteriormente los cuadrados ABP Q, CARS y
BCT U . Luego se trazan los paralelogramos AQA0 R, CSC 0 T y BU B 0 P .

a) Sean A00 , B 00 , C 00 los centros de los cuadrados BCT U , CARS, ABP Q, respectivamente.
Demuestre que estos centros están sobre los lados del 4A0 B 0 C 0 .
b) Demuestre que AA00 , BB 00 , CC 00 concurren.

18. (*) Se dibujan cuadrados exteriores a los lados de un paralelogramo, demuestre que:

a) El cuadrilátero determinado por los centros de esos cuadrados es un cuadrado.


b) Las diagonales de ese cuadrado son concurrentes con las del paralelogramo.

UES-FCNM-Escuela de Matemática-Licenciatura en Matemática y Licenciatura en Estadı́stica


pág. 47
Geometrı́a I. 5 Cuadriláteros

19. (*) Dado un 4ABC, se construyen exteriormente los triángulos rectángulo isósceles 4ACP
y 4BCQ, con AC y BC como hipotenusas. Si M es el punto medio de AB, demuestre que
el 4M P Q también es un triángulo rectángulo isósceles.

UES-FCNM-Escuela de Matemática-Licenciatura en Matemática y Licenciatura en Estadı́stica


pág. 48
Geometrı́a I. 6 La Circunferencia

6. La Circunferencia
6.1. Elementos de la circunferencia
Una circunferencia es el lugar geométrico de puntos que equidistan de un punto dado, llama-
do el centro de la circunferencia; la distancia de cada punto de la circunferencia al centro es el radio.

Por otra parte, todos los puntos que están a una distancia del centro menor o igual al radio
forman el cı́rculo; estos puntos quedan “al interior” o sobre la circunferencia.

Si A y B son dos puntos de una circunferencia, el segmento de recta AB define una cuerda;
en particular, si el centro de la circunferencia pertenece a la cuerda, ésta es llamada diámetro.
Es importante mencionar que para cada punto de la circunferencia existe exactamente un punto
diametralmente opuesto.

En la figura 54, se tiene una circunferencia de centro O y radio r = OA = OB = OA0 ; AB y


AA0 son cuerdas, pero AA0 es también diámetro, i.e, A0 es diametralmente opuesto a A y viceversa.
Observe que por la desigualdad triangular aplicada al triángulo isósceles 4AOB

Figura 54:

AB < AO + BO
= r+r
= AA0

Si A es un punto fijo, esta desigualdad es válida para cualquier punto B sobre la circunferencia
(excepto cuando B = A0 lo cual implica AB = AA0 ). Esto quiere decir que el diámetro es la mayor
de todas las cuerdas.

6.2. Ángulos en la circunferencia


A las porciones de circunferencia que quedan entre dos puntos ubicados en la circunferencia,
se les llama arcos de circunferencia; note que dos puntos sobre una circunferencia definen dos
arcos de circunferencia. También, si un ángulo tiene vértice sobre el centro de la circunferencia y
está formado por dos radios, será llamado ángulo central ; de nuevo, ∠AOB hace referencia a dos
ángulos, cuya suma es 360, y subtienden respectivamente a uno de los arcos AB. Finalmente, si un

UES-FCNM-Escuela de Matemática-Licenciatura en Matemática y Licenciatura en Estadı́stica


pág. 49
Geometrı́a I. 6 La Circunferencia

ángulo tiene el vértice sobre la circunferencia y está formado por dos cuerdas, será llamado ángulo
inscrito; en la figura anterior, ∠AA0 B es un ángulo inscrito que subtiende al arco AB.

Teorema: El ángulo central es el doble del ángulo inscrito que subtiende el mismo arco.

Demostración: Considere la figura 55, se demostrará que ∠AOB = 2∠AP B en los tres casos
mostrados. En la circunferencia de la izquierda, sea P 0 el punto diametralmente opuesto a P ;
observe que 4AP O y 4BP O son triángulos isósceles, y por el teorema del ángulo externo se tiene

∠AOB = ∠AOP 0 + ∠BOP 0


= (∠AP O + ∠OAP ) + (∠BP O + ∠OBP )
= 2∠AP O + 2∠BP O
= 2 (∠AP O + ∠BP O)
= 2∠AP B

Figura 55:

El caso de la circunferencia del medio es más sencillo y se deja como ejercicio para el lector.
Para la circunferencia de la derecha, el trabajo es análogo y sólo cambia en un pequeño arreglo
algebraico

∠AOB = ∠BOP 0 − ∠AOP 0


= (∠BP O + ∠OBP ) − (∠AP O + ∠OAP )
= 2∠BP O − 2∠AP O
= 2 (∠BP O − ∠AP O)
= 2∠AP B

Corolario: Todos los ángulos inscritos que subtienden el mismo arco son iguales (Ver figura
56). En particular, los ángulos internos son iguales a 90° si subtienden a una semicircunferencia.

Demostración: Todos los ángulos mostrados en la figura 56 son iguales a la mitad del ∠AOB,
y por tanto, son iguales entre sı́. En particular, si AB fuera un diámetro, ∠AOB = 180° y por
tanto ∠AP B = 90°. 14

Hay un par de ángulos más que son importantes: Si un punto P es interno a la circunferencia,
el ángulo de vértice P formado por dos cuerdas que pasan por P se llama ángulo interior. De
14
Observe que en cualquier triángulo rectángulo, el punto medio de la hipotenusa equidista de los tres vértices.

UES-FCNM-Escuela de Matemática-Licenciatura en Matemática y Licenciatura en Estadı́stica


pág. 50
Geometrı́a I. 6 La Circunferencia

Figura 56:

forma similar, si P es exterior y dos cuerdas de la circunferencia (al prolongarse) pasan por P , el
ángulo con vértice P es llamado ángulo exterior.

Dejamos como ejercicio demostrar el siguiente teorema:

Teorema: Los ángulos interior y exterior mostrados en la figura 57 cumplen las fórmulas
siguientes:
∠BOD + ∠AOC
∠AQC =
2
∠BOD − ∠AOC
∠AP C =
2

Figura 57:

6.3. Cuadriláteros Concı́clicos


Ahora suponga que sobre una circunferencia se ubican cuatro puntos A, B, C, D, como se
muestra en la figura 58. Al cuadrilátero ABCD se le llama cuadrilátero cı́clico o concı́clico. Observe
que
α β
∠ABC + ∠CDA = + = 180◦ .
2 2

UES-FCNM-Escuela de Matemática-Licenciatura en Matemática y Licenciatura en Estadı́stica


pág. 51
Geometrı́a I. 6 La Circunferencia

Figura 58:

Y análogamente ∠DAB +∠BCD = 180°. Esto significa que si ABCD es un cuadrilátero cı́clico
y convexo, entonces los ángulos opuestos son suplementarios. También, es posible demostrar por
contradicción el recı́proco de este resultado: si suponemos que ABCD es tal que ∠B + ∠D = 180
pero no es cı́clico, se define el punto D0 como la otra intersección de AD con el circuncı́rculo del
4ABC, y como ABCD0 es cı́clico (por construcción) entonces ∠B +∠D0 = 180, luego, ∠D = ∠D0 ,
lo cual implica la contradicción CD k CD0 (rectas paralelas que se cortan en C). Ası́, se ha de-
mostrado el siguiente teorema:

Teorema: El cuadrilátero convexo ABCD es un cuadrilátero cı́clico si y sólo si

∠A + ∠C = 180◦ = ∠B + ∠D

También, otro criterio muy útil y cuya demostración también se basa en el corolario anterior
es

Teorema: El cuadrilátero convexo ABCD es un cuadrilátero cı́clico si y sólo si se cumple


alguna de las siguientes igualdades

∠ABD = ∠ACD
∠BCA = ∠BDA
∠BAC = ∠BDC
∠CAD = ∠CBD

Es importante recalcar que NO todo cuadrilátero puede ser inscrito en una circunferencia; por
ejemplo, un paralelogramo no será cı́clico a menos que sea rectángulo.

Teorema de Miquel: Si D, E, F son tres puntos cualesquiera en los lados BC, CA, AB del
triángulo ABC, entonces las circunferencias que pasan por las tercias de puntos B, D, F ; C, E, D; A, F, E;
tienen un punto en común. Ver figura 59
Prueba: (Ejercicio, Use concı́clicos )
Teorema: La Recta de Simson-Wallace. Sean X, Y y Z los pies de las alturas trazadas
desde un punto P en el circuncı́rculo del 4ABC hacia AB, BC y CA, respectivamente, son
colineales.

UES-FCNM-Escuela de Matemática-Licenciatura en Matemática y Licenciatura en Estadı́stica


pág. 52
Geometrı́a I. 6 La Circunferencia

Figura 59:

6.4. Rectas y Circunferencias tangentes a una circunferencia


Dada una circunferencia, una recta puede ser tangente o secante a la circunferencia, depen-
diendo si la corta en uno o dos puntos, respectivamente; en cualquier otro caso, se dice que la
recta no corta a la circunferencia.15

Sea l una recta secante a la circunferencia que corta a la circunferencia en A y B (A 6= B);


como el 4AOB es isósceles, ∠OAB < 90. Recı́procamente, si por A se traza una recta l tal que
uno de los ángulos que forma con OA es menor que 90, se puede construir un punto B sobre l tal
que ∠OAB = ∠ABO < 90 y A 6= B (basta proyectar O sobre l y luego reflejar A con respecto a
este punto, el resultante es el punto B); entonces el 4AOB es isósceles, por lo que OA = r = OB,
i.e. B pertenece a la circunferencia y por tanto l corta a la circunferencia en dos puntos distintos. Ası́

Teorema: Una recta l corta a una circunferencia de centro O en dos puntos distintos A y B
si y sólo si un ángulo entre l y OA es agudo.

Corolario: Si l es una recta tangente en A a una circunferencia de centro O, ninguno de los


ángulos entre l y OA puede ser agudo, y por tanto l ⊥ OA.

A partir de este resultado se prueban otros resultados muy conocidos y útiles, que dejamos de
ejercicios para el lector.

Teorema: Dado un punto P externo a una circunferencia de centro O, si P A y P B son seg-


mentos tangentes a la circunferencia en A y B, respectivamente, entonces el cuadrilátero P AOB
es cı́clico y bisósceles.

Corolario: Dado un punto P externo a una circunferencia de centro O, la circunferencia de


diámetro P O corta a la circunferencia dada en dos puntos A y B tales que P A y P B son rectas
tangentes.

Definición: El ángulo semi-inscrito en una circunferencia es aquel que se forma con una cuer-
da y la recta tangente en alguno de los extremos de la cuerda.

15
Cuando la recta es tangente a la circunferencia puede considerarse como un caso muy peculiar en el cual los
“dos” puntos de corte coinciden.

UES-FCNM-Escuela de Matemática-Licenciatura en Matemática y Licenciatura en Estadı́stica


pág. 53
Geometrı́a I. 6 La Circunferencia

Teorema: La media del ángulo semi-inscrito definido por la cuerda AB es igual a la medida
de un ángulo inscrito que subtiende al arco AB.

Demostración: Considere la figura 60. Como AP BO es cı́clico, entonces ∠P AB = ∠P OB;


además, como P O es la mediatiz de AB, ∠P OB = ∠P OA, por lo que
∠AOB
∠P AB = = ∠AQB
2

Figura 60:

Por otra parte, dada una circunferencia, otra circunferencia puede ser secante o tangente a la
primera, dependiendo si la corta en uno o dos puntos, respectivamente; en cualquier otro caso se
dice que las circunferencias no se cortan.16

Además, dos circunferencias pueden posicionarse una dentro de la otra, y claramente, la cir-
cunferencia de radio mayor es la externa mientras que otra es la interna; particularmente, si las
circunferencias tienen el mismo centro se llaman concéntricas. Finalmente, combinando estas de-
finciones se tienen las circunferencias tangentes exteriormente y las tangentes interiormente.

Teorema: Dadas dos circunferencias de centros O1 y O2 que se cortan en dos puntos distintos
A y B, se cumple que O1 O2 ⊥ AB.

Teorema: Si dos circunferencias de centros O1 y O2 son tangentes en A, se cumple que O1 , A


y O2 están alineados.

Teorema:
a) Dos circunferencias, una dentro de la otra, no tienen rectas tangentes en común.

b) Dos circunferencias tangentes interiormente tienen una recta tangente común.

c) Dos circunferencias secantes (en dos puntos distintos) tienen dos rectas tangentes en común.

d) Dos circunferencias tangentes exteriormente tienen tres rectas tangentes en común.

e) Dos circunferencias no secantes y tal que ninguna contiene a la otra, tienen cuatro rectas
tangentes en común.
16
También acá puede considerarse a las circunferencias tangentes como un caso especial de circunferencias secantes
en el cual los puntos de corte coinciden.

UES-FCNM-Escuela de Matemática-Licenciatura en Matemática y Licenciatura en Estadı́stica


pág. 54
Geometrı́a I. 6 La Circunferencia

6.5. Problemas

1. Si el ∠M P Q = 20, determine el valor del ∠QON en la figura


adjunta.

2. Dado un ángulo inscrito BAC, y su ángulo central BOC, se sabe que ∠BAC +∠BOC = 180°.
Calcular el ∠OBC.

3. En la figura 61, BCDO es un rombo. Determine el valor del ángulo θ y la medida de las
diagonales de BCDO si el radio de la circunferencia mide 6.

Figura 61:

4. Un cuadrilátero cı́clico ABCD satisface ∠ABC = 2∠CDA = θ. Calcule θ.

UES-FCNM-Escuela de Matemática-Licenciatura en Matemática y Licenciatura en Estadı́stica


pág. 55
Geometrı́a I. 6 La Circunferencia

5. En la figura 62. Calcule el valor del ∠P QR.

Figura 62:

6. En la figura adjunta, el ∠AF E = 100° y el ∠BCD =


150°. Calcule el ∠AGB.

7. Dado un ángulo ∠AOB, se trazan dos rectas l y m perpendiculares a los lados del ángulo
en A y B respectivamente. Si P es el punto de corte de l y m, demuestre que A, B, O, P se
ubican sobre una misma circunferencia.

8. En la figura 63 se ha tomado un punto C sobre la circunferencia de centro O; AC y BC


cortan a la segunda circunferencia en D y E respectivamente. Probar que OC ⊥ DE.

Figura 63:

UES-FCNM-Escuela de Matemática-Licenciatura en Matemática y Licenciatura en Estadı́stica


pág. 56
Geometrı́a I. 6 La Circunferencia

9. (*) Dada la figura 64, demuestre que AB k A0 B 0 .

Figura 64:

10. En la figura 65 CR es una recta tangente en C, demuestre que AB k CR.

Figura 65:

11. Dos circunferencias Γ1 y Γ2 son tangentes (interior o exteriormente) en P (Ver figura 66). Dos
rectas que pasan por P cortan a Γ1 y Γ2 en A y C, y en B y D, respectivamente. Demuestre
que AB k CD.

Figura 66:

UES-FCNM-Escuela de Matemática-Licenciatura en Matemática y Licenciatura en Estadı́stica


pág. 57
Geometrı́a I. 6 La Circunferencia

12. (*) Dos circunferencias de centros O1 y O2 son tangentes (interna o externamente) en un


punto P ; por este punto se traza una recta que corta nuevamente a la circunferencias en A
y B, respectivamente. Demuestre que AO1 k BO2 .

13. Dos circunferencias son tangentes externamente en el punto A. Una tangente exterior común
toca a una circunferencia en B y a la otra en C. Demostrar que ∠BAC = 90°.

14. En la figura 67, DE es tangente en D, y C es el punto medio del arco AD. Encuentre el
valor del ángulo seminscrito ADE.

Figura 67:

15. Determine el valor del ∠DCF , sabiendo BE es tangente en el punto D a la circunferencia


de centro O. Ver Figura 68.

Figura 68:

16. Si el ∠AEB = 30, ∠ADE = 20 y ∠ACE = 35, calcule el ∠AF B. Véase figura 69.

UES-FCNM-Escuela de Matemática-Licenciatura en Matemática y Licenciatura en Estadı́stica


pág. 58
Geometrı́a I. 6 La Circunferencia

Figura 69:

17. Dada una circunferencia de diámetro BC, se toma un punto P en la prolongación de BC, y
se traza la tangente AP . Si AP = AB y O es el centro de la circunferencia, demuestre que
el 4AOC es equilátero.

18. (*) Dadas dos circunferencias una fuera de la otra, demuestre que las tangentes comunes
externas forman segmentos iguales; análogamente, las tangentes comunes internas forman
segmentos iguales.

19. (*) Teorema de Pithot. Demuestre que en todo cuadrilátero inscribible, la suma de lados
opuestos es igual.

20. (*) Teorema de Steiner. En todo cuadrilátero exinscrito a una circunferencia, la diferencia
de las longitudes de lados opuestos es igual.

21. En la figura 70, AB es una cuerda y por D se traza una recta tangente a la circunferencia
paralela a AB. Demuestre que CD es bisectriz del ∠ACB.

Figura 70:

22. (X OMCC - P2, Aarón) Sea ABCD un cuadrilátero concı́clico con diámetro AC, y sea O el
centro de su circunferencia. Se construyen los paralelogramos DAOE y BCOF . Demuestre
que si E y F están sobre la circunferencia entonces ABCD es rectángulo.

UES-FCNM-Escuela de Matemática-Licenciatura en Matemática y Licenciatura en Estadı́stica


pág. 59
Geometrı́a I. 6 La Circunferencia

23. Cuatro cilindros de diámetro 1 están pegados apretadamente por


una cuerda muy fina, como en la figura adjunta. Demostrar que la
cuerda tine longitud 4 + π. Demostrar también que el área som-
breada entre los cilindros es 1 − π4 .

24. En la figura 71, ABCD es un trapecio isósceles con AB k CD y DA = BC = 2; tomando DA


y BC como diámetros, se construyen dos circunferencias tangentes. Si DC = 3AB, calcule
el área del trapecio.

Figura 71:

25. La figura 72 está formada por un paralelogramo y dos circunferencia tangentes entre sı́ y
tangentes a tres lados del paralelogramo. Sabiendo que el radio de las mismas mide la cuarta
parte del lado menor del paralelogramo, calcule la razón entre el lado mayor del paralelogramo
y el radio de las circunferencias.

Figura 72:

26. (*) Sea ABC un triángulo, y sean L y N las intersecciones de la bisectriz del ángulo A con
el lado BC y el circuncı́rculo de ABC respectivamente. Construimos la intersección M del
circuncı́rculo de ABL con el segmento AC. Prueba que los triángulos BM N y BM C tienen
la misma área.

27. (*) Sea AB el diámetro de una semicircunferencia. Se colocan los puntos M y K sobre la
semicircunferencia y sobre AB, respectivamente.17 Sea P el centro de la circunferencia que
pasa por A, K y M ; sea Q el centro de la circunferencia que pasa por B, K y M . Demuestre
que M P KQ es concı́clico.
17
M y K son distintos de A y B.

UES-FCNM-Escuela de Matemática-Licenciatura en Matemática y Licenciatura en Estadı́stica


pág. 60
Geometrı́a I. 6 La Circunferencia

28. En la figura 73, ABCDEF es un hexágono regular y las circunferencias de centro en los
vértices son tangentes dos a dos. Si las circunferencias sobre los vértices B, D, F son iguales,
demuestre que las circunferencias restantes son iguales.

Figura 73:

29. (*) Las circunferencias Γ1 y Γ2 se cortan en los puntos A y B. Por el punto A se traza una recta
que corta nuevamente a las circunferencias Γ1 y Γ2 en los puntos C y D, respectivamente.
Por los puntos C y D se trazan tangentes a las circunferencias, las cuales se cortan en el
punto M . Demuestra que M CBD es cı́clico.
30. (*) El 4ABC cumple que ∠A = 90° y AB = AC. Se toma un punto E del segmento AB,
se construye interiormente un triángulo equilátero AEF . EF corta BC en I, y se construye
exteriormente un triángulo equilátero BIJ. Encuentre ∠EJB.
31. (*) En la figura 74, se sabe que ∠AO1 B − ∠AO2 B = 70◦ y además la tangente EB forma el
triángulo isósceles ABE, con AB = AE. Encuentre ∠EBC.

Figura 74:

32. (*) Dos circunferencias Γ1 y Γ2 se cortan en A y B. Una recta por A corta a Γ1 y Γ2 en C


y D, respectivamente, y la paralela a CD por B corta Γ1 y Γ2 en E y F , respectivamente.
Demuestre que 4CDB ≡ 4EAF .

UES-FCNM-Escuela de Matemática-Licenciatura en Matemática y Licenciatura en Estadı́stica


pág. 61
Geometrı́a I. 6 La Circunferencia

33. (*) Sea P un punto exterior al cuadrado ABCD tal que ∠AP C = 90◦ , Q es la intersección
de AB y P C, y R el pie de la perpendicular por Q a CA. Demuestre que P , R y D están
alineados.

34. En la figura 75, ABCD es un trapecio rectángulo tal que la circunferencia de diámetro AB
(y centro O) es tangente a CD. Demostrar que O pertenece a la circunferencia de diámetro
CD y que esta circunferencia es tangente a BA.

Figura 75:

35. El 4ABC es rectángulo en C, la circunferencia de centro O es tangente a cada uno de los


lados del 4ABC en los puntos P , Q y R (como se muestra en la figura 76), y se cumple que
AP = 20 y BP = 6. Calcule OP .

Figura 76:

36. Los vértices A y B de un triángulo equilátero 4ABC están sobre una circunferencia de
radio 1 y el vértice C está en el interior de la circunferencia. Un punto D (distinto de B)
que esta en la circunferencia es tal que AD = AB. La recta DC corta por segunda vez a la
circunferencia en E. Encuentre la longitud del segmento CE. Ver figura 77.

37. En la figura 78 se muestran tres semicircunferencias, una de diámetro AB (de centro O y


radio r), otra de diámetro AO y la última de diámetro OB. Determine la razón entre el radio
de la circunferencia tangente a estas tres semicircunferencias y r.

38. El segmento AB es diámetro de un semicı́rculo con centro en O. Un cı́rculo con centro en P


es tangente a AB en O y también al semicı́rculo. Otro cı́rculo con centro en Q es tangente a
AB, al semicı́rculo y al cı́rculo de centro en P . Si AB = 2, ¿cuál es el radio del cı́rculo con
centro en Q?

UES-FCNM-Escuela de Matemática-Licenciatura en Matemática y Licenciatura en Estadı́stica


pág. 62
Geometrı́a I. 6 La Circunferencia

Figura 77:

Figura 78:

Figura 79:

39. (*) (OIM 2002, P-4) En un triángulo escaleno ABC se traza la bisectriz interior BD, con D
sobre AC. Sean E y F puntos sobre la recta BD tales que (AE k CF ) ⊥ BD, y sea M el
punto sobre el lado BC tal que DM ⊥ BC. Demuestre que ∠EM D = ∠DM F .

40. (*) (OMCC 2003, P-2) Sea S una circunferencia y AB un diámetro de ella. Sea t la recta
tangente a S en B y considere dos puntos C y D en t tales que B este entre C y D. Sean E
y F las intersecciones de S con AC y AD y sean G y H las intersecciones de S con CF y
DE. Demuestre que AH = AG.

41. (*) (The 59th Romanian Mathematical Olympiad District Round) Considere un cuadrado
ABCD y un punto E sobre el lado AB. La diagonal AC corta al segmento DE en el punto
P . La perpendicular por P a DE corta al lado BC en F . Probar que EF = AE + CF .

42. (*) Teorema de Arquı́medes: En la figura 80, la región delimitada por tres semicircunfe-
rencias mutuamente tangentes, es conocida como cuchilla de zapatero o árbelos. Demostrar
que las circunferencias sombreadas son congruentes.

UES-FCNM-Escuela de Matemática-Licenciatura en Matemática y Licenciatura en Estadı́stica


pág. 63
Geometrı́a I. 6 La Circunferencia

Figura 80: Teorema de Arquı́medes.

43. Demuestre que las rectas de Simson-Wallace de dos puntos diametralmente opuestos son
perpendiculares.

UES-FCNM-Escuela de Matemática-Licenciatura en Matemática y Licenciatura en Estadı́stica


pág. 64
Geometrı́a I. 7 Semejanza de Triángulos.

7. Semejanza de Triángulos.
7.1. Proporcionalidad
Definición
1. Razón: se llama razón, al cociente de dos cantidades, expresadas en la misma magnitud, por
ejemplo ab .

2. Proporción: se llama proporción a la igualdad de dos razones. Por ejemplo ab = dc , 18 a los


términos a y d se les llama extremos y los términos b y c se les llama medios, al término d se
le llama cuarta proporcional entre a, b y c en este orden.
Propiedades de las proporciones:
a c
1. = si y sólo si a· c = b· d.
b d
a c b d a b
2. = si y sólo si = o = .
b d a c c d
a c a±b c±d
3. = si y sólo si = .
b d b d
a c a+b c+d
4. = si y sólo si = .
b d a−b c−d
Proporción Armónica.
Cuaterna Armónica: Los puntos colineales y consecutivos A, B, C y D forman una cuaterna armóni-
AB AD
ca si y solamente si BC = CD .
Propiedades
1. AB > BC.
2 1 1
2. Relación de Descartes AC
= AB
+ AD
.

3. Relación de Newton. Si O es punto medio de AC entonces: (OC)2 = OB.OD.


AB AD
4. Si los segmentos determinados por la cuaterna armónica, verifican la relación BC
= n CD ,
donde n > 0 entonces n+1
AC
n
= AB 1
+ AD .

Sección Áurea de un segmento


Se dice que un punto C divide al segmento AB en la proporción áurea cuando, siendo AC la parte
mayor en la que AB queda dividido por el punto C, se cumple: AB AC
AC
= CB . A la parte mayor en
la que AB queda dividido por C se la llama segmento áureo del segmento AB. El problema de la
división áurea de un segmento fue resuelto por Euclides en los Elementos II. 11, y desde entonces
ha sido asunto de interés para los matemáticos de todos los tiempos.
Propiedades

5+1
1. AB = AC 2
.
18
En algunos textos de geometrı́a se utiliza la notación de proporción ası́ a : b :: c : d que se lee “a es a b como c
es a d”.

UES-FCNM-Escuela de Matemática-Licenciatura en Matemática y Licenciatura en Estadı́stica


pág. 65
Geometrı́a I. 7 Semejanza de Triángulos.


5−1
2. AC = AB 2

5+1
3. Número Áureo Φ = 2

7.2. Teorema de Thales


Definición
PA
1. Un punto P ∈ AB divide al segmento AB en una razón dada r, si PB
= r.

2. Sean AB y CD dos segmentos, y sean P ∈ AB y Q ∈ CD, decimos que P y Q dividen a


CQ
AB y CD en segementos proporcionales si PAPB = QD .

Figura 81:

Teorema de Thales. Si tres paralelas cortan a dos secantes entonces los segmentos que de-
terminan en ellas son proporcionales. 19

Antes de demostrar el Teorema de Thales, se enunciarán dos teoremas que a pesar de su apa-
rente sencillez es de mucha utilidad en problemas que involucran Áreas y Proporcionalidad.

Lema 1. Sea AB k CD. Demuestre que: (ABC) = (ABD).

Lema 2. Sea P un punto sobre el lado AB (o su prolongación) del 4ABC. Pruebe que:

AP (AP C)
=
PB (P BC)

A continuación se enuncian los pasos a seguir en la demostración del teorema de Thales.

Demostración. Sean AA0 , BB 0 y CC 0 rectas paralelas que cortan a dos secantes en los puntos
A, A0 , B, B 0 , C, C 0 respectivamente (ver figura 82).
Pruebe que:
AB (ABB 0 )
1. =
BC (BCB 0 )
19
El teorema de Thales puede enunciarse de manera general como sigue: Si tres o más paralelas cortan a dos o
más secantes entonces los segmentos que determinan en ellas son proporcionales.

UES-FCNM-Escuela de Matemática-Licenciatura en Matemática y Licenciatura en Estadı́stica


pág. 66
Geometrı́a I. 7 Semejanza de Triángulos.

Figura 82: Teorema de Thales

A0 B 0 (A0 B 0 B)
2. = .
B0C 0 (B 0 C 0 B)
3. (ABB 0 ) = (A0 B 0 B) y (BCB 0 ) = (B 0 C 0 B).

Con ayuda de las igualdades demostradas concluya que:

AB A0 B 0
= 0 0.
BC BC
Observación Importante: Utilice las propiedades de las proporciones para demostrar las equi-
valencias siguientes (interprételas geométricamente):

AB A0 B 0 AC A0 C 0 AC A0 C 0
= 0 0 ⇔ = 0 0 ⇔ = 0 0
BC BC AB AB BC BC
Sea4ABC un triángulo, sabemos que su area puede calcularse al multiplicar la longitud de
uno de sus lados por la longitud de la altura correspondiente a ese lado. Si denotamos por a la
longitud de un lado del 4ABC y ha la longitud de la altura correspondiente, el area del 4ABC
se denota por (ABC) y es igual a:
a · ha
2
Ahora estamos en condiciones de probar el siguiente lema:
Lema:Si dos triangulos tienen una misma altura entonces la razon entre sus areas es igual a
la razon de las bases donde se levanta la altura comun.
Demostracion:Sean 4ABC y 4A0 B 0 C 0 triangulos con alturas iguales h. Sean D y D0 los pies
de las alturas trazadas de los vertices A y A0 a los lados BC y B 0 C 0 respectivamente. Entonces:

BC · h 0 0 0 B0C 0 · h
(ABC) = (A B C ) =
2 2
La razon entre las areas es:
BC·h
(ABC) 2 BC
= B 0 C 0 ·h
=
(A0 B 0 C 0 ) 2
B0C 0
Como queriamos demostrar.
El siguiente lema se puede probar de manera analoga al anterior:
Lema:Si dos triangulos tienen una base igual entonces la razon de sus areas es igual a la razon
entre las alturas que se levantan sobre la base igual.

UES-FCNM-Escuela de Matemática-Licenciatura en Matemática y Licenciatura en Estadı́stica


pág. 67
Geometrı́a I. 7 Semejanza de Triángulos.

Corolario (Teorema de Thales en el triángulo). Toda recta paralela a un lado de un triángulo


y que corte a los otros dos lados, divide a estos lados en segmentos proporcionales.

Recı́proco del Teorema de Thales. Si tres rectas cortan a dos secantes en segmentos pro-
porcionales y dos de estas rectas son paralelas entonces las tres rectas son paralelas.

Demostración. Sean AA0 , BB 0 y CC 0 rectas que cortan a dos secantes en los puntos A, A0 ,
AB A0 B 0
B, B 0 , C, C 0 respectivamente, tales que AA0 k CC 0 y = 0 0 . Por el punto B tracemos
BC BC
una recta paralela a AA0 , la cual interseca a A0 C 0 en el punto D (ver figura 83). Entonces, por el
AB A0 D A0 B 0 A0 D
Teorema de Thales se tiene que: = . De donde, = , ası́ por las propiedades
0 0 0 0
BC DC 0 B0C 0 DC 0
AC AC
de las proporciones 0 0 = 0
, por lo que B 0 C 0 = B 0 D + DC 0 = DC 0 y por tanto B 0 D = 0, o
BC DC
equivalentemente B 0 = D y por lo tanto, BB 0 k AA0 .

Figura 83: Recı́proco del Teorema de Thales

Corolario (Recı́proco del Teorema de Thales en el triángulo.) Si una recta intercepta


dos lados de un triángulo en segmentos proporcionales entonces la recta es paralela al tercer lado
del triángulo.
Teorema:Sea 4ABC un triangulo y sean D y E puntos en los lados AB y AC respectivamente.
Si se cumple que:
AB AC
=
AC AE
entonces DE es paralela a BC.
Supongamos que, por el contrario DE no es paralela a BC. Sea C 0 un punto en AC, distinto
0
de C, tal que BC 0 es paralela a DE, entonces por el teorema de Thales, AD
AB
= AC
AE
. Pero ademas,
AB AC
se cumple por hipotesis del problema, que AD = AE . Podemos entonces decir que
AB · AE
AC = = AC 0
AD

AC = AC 0
Por tanto, los puntos C y C 0 deben ser iguales y DE es paralela a BC.
Teorema: Consideremos tres rectas paralelas y dos rectas transversales a estas como se muestra
AB
en la figura. Tenemos que si AD, BE y CF son paralelas entonces BC = DE
EF
. Reciprocamente, si
AB DE
BC
= EF
y dos de las rectas AD, BE o CF son paralelas entonces las rectas son paralelas.
Demostracion. Diremos que G es el punto de interseccion de AF con BE. Si aplicamos el
teorema de Tales y su reciproco en los triangulos 4ACF y 4F DA, vemos que las rectas AD, BE,
CF son paralelas si y solo si

UES-FCNM-Escuela de Matemática-Licenciatura en Matemática y Licenciatura en Estadı́stica


pág. 68
Geometrı́a I. 7 Semejanza de Triángulos.

AB AG FG FE
BC
= GF
y GA
= ED
AB
Luego, son paralelas si y solo si BC = FAG
G
= DE
EF
.
AB
Ahora, supongamos que BE y CF son paralelas y que la otra recta AD cumple que BC = DE
EF
.
Definimos G como el punto de interseccion de AF con BE. Como BE y CF son paralelas, se
AB AG AB
cumple que BC = GF . Ahora bien, como BC = DE
EF
AG
tenemos que, GF = DE
EF
. Luego, el primer
teorema de Thales, GE es paralela a AD y de ahi que BE es tambien paralela AD.

7.3. Criterios de Semejanza de Triángulos


Triángulos semejantes. Decimos que el 4ABC es semejante al 4A0 B 0 C 0 (Ver figura 84), lo
cual denotamos ası́ ABC ∼ A0 B 0 C 0 , si:
AB AC BC
= =
A0 B 0 A0 C 0 B0C 0
y
∠BAC = ∠B 0 A0 C 0 , ∠ABC = ∠A0 B 0 C 0 , ∠ACB = ∠A0 C 0 B 0 .

Figura 84: Definición de Semajanza de Triángulos.

En los tres teoremas que se muestran a continuación (los cuales son una consecuencia directa
del Teorema de Thales) se establecen las condiciones mı́nimas para demostrar que dos triángulos
son semejantes, a los cuales denominaremos: Criterios de Semejanza de Triángulos.

Primer criterio de semejanza de triángulos: Angulo-Angulo A-A. Si dos ángulos de


un triángulo son congruentes con dos ángulos de otro triángulo, entonces los dos triángulos son
semejantes.
Demostración. Supongamos que en el 4ABC y 4A0 B 0 C 0 se tiene que ∠ABC = ∠A0 B 0 C 0 y
∠ACB = ∠A0 C 0 B 0 , entonces ∠BAC = ∠B 0 A0 C 0 (Por la suma de ángulos internos en un triángulo).
Sea D ∈ AB y E ∈ AC tales que AD = A0 B 0 y AE = A0 C 0 , dado que ∠DAE = ∠BAC =
∠B 0 A0 C 0 , se sigue por L-A-L que 4ADE = 4A0 B 0 C 0 , por consiguiente ∠ADE = ∠A0 B 0 C 0 =
∠ABC, de donde DE k BC (por ser iguales los ángulos correspondientes) y por el teorema de
Thales
AB AC
=
AD AE
y por consiguiente
AB AC
0 0
= 0 0 (2)
AB AC

UES-FCNM-Escuela de Matemática-Licenciatura en Matemática y Licenciatura en Estadı́stica


pág. 69
Geometrı́a I. 7 Semejanza de Triángulos.

Sea F ∈ BC tal que DF k AC, entonces F C = DE = B 0 C 0 (porque DECF es paralelogramo


y por ser 4ADE = 4A0 B 0 C 0 ) y por el teorema de Thales

BA BC
=
DA FC
o lo que es lo mismo
AB BC
= (3)
A0 B 0 B0C 0
Luego, de (1) y (2) se tiene que:

AB AC BC
0 0
= 0 0 = 0 0.
AB AC BC
Ası́, se ha demostrado que los tres pares de ángulos son congruentes y los tres pares de lados son
proporcionales, por lo tanto, 4ABC ∼ 4A0 B 0 C 0 .

Segundo criterio de semejanza de triángulos: L-A-L. Si un ángulo de un triángulo es


congruente con otro ángulo de otro triángulo y los lados que comprenden al ángulo en el primer
triángulo son respectivamente proporcionales a los lados que comprende al ángulo en el segundo
triángulo, entonces los dos triángulos son semejantes.
AB AC
Demostración. Suponga que el ∠BAC = ∠B 0 A0 C 0 y que 0 0 = 0 0 . Considere los
AB AC
puntos D y E, como en la demostración del teorema anterior. Entonces por el criterio L-A-L,
4ADE = 4A0 B 0 C 0 , de lo cual se deduce que ∠ADE = ∠A0 B 0 C 0 . Por otra parte tenemos que:
AB AC
= , y al aplicar el recı́proco del teorema de Thales, se puede afirmar que DE k BC, de
AD AE
lo cual a su vez se deduce que ∠ADE = ∠ABC, por ángulos correspondientes entre paralelas.
Finalmente por transitividad se concluye que ∠ABC = ∠A0 B 0 C 0 . Por lo tanto, 4ABC ∼ 4A0 B 0 C 0
(Por el criterio A-A.)

Tercer criterio de semejanza de triángulos: L-L-L. Si los tres lados de un triángulo son
respectivamente proporcionales a los tres lados de otro triángulo, entonces los dos triángulos son
semejantes.
AB AC BC
Demostración. Por hipótesis se tiene que: 0 0 = 0 0 = 0 0 y como antes sean D y E
AB AC BC
puntos sobre AB y AC respectivamente tales que AD = A0 B 0 y AE = A0 C 0 . Entonces por el
recı́proco del teorema de Thales se tiene que DE k BC y por consiguiente el ∠ABC = ∠ADE
AB BC
y el ∠ACB = ∠AED, de donde 4ABC ∼ 4ADE (por el criterio A-A). Por ende = ,
AD DE
BC BC
luego por transitividad = 0 0 , de donde DE = B 0 C 0 . En consecuencia 4ADE = 4A0 B 0 C 0
DE BC
(por el criterio L-L-L), de lo cual se sigue que ∠A0 B 0 C 0 = ∠ADE y ∠A0 C 0 B 0 = ∠AED, y por
transitividad ∠A0 B 0 C 0 = ∠ABC y ∠A0 C 0 B 0 = ∠ACB =. Por lo tanto, 4A0 B 0 C 0 ∼ 4ABC (Por
el criterio A-A.)

7.4. Potencia de Punto


Proposición.

UES-FCNM-Escuela de Matemática-Licenciatura en Matemática y Licenciatura en Estadı́stica


pág. 70
Geometrı́a I. 7 Semejanza de Triángulos.

Si dos cuerdas AB y CD de una circunferencia se intersectan en un punto P , entonces P A·P B =


P C · P D.

Demostración. Si P es un punto sobre la circunferencia está claro que ambos productos


son 0 y por tanto iguales. Si P es un punto dentro de la circunferencia entonces el cuadrilátero
ADBC es cı́clico con los ángulos ∠BAD = ∠BCD y ∠ADC = ∠ABC y por tanto los triángulos
4AP D y 4BP C y por tanto se cumple las proporciones PP CA = PP DB
y despejando obtenemos que
P A · P B = P C · P D.

Si P es un punto fuera de la circunferencia el cuadrilátero ABDC es ciclico y los ángulos ∠CAP


y ∠BDP son iguales y los triángulos 4P AC y 4P BD son triángulos semejantes con lo que se
cumple de nuevo en este caso que PP CA = PP D
B
y por tanto, P A · P B = P C · P D.

Proposición 2. Si A, B y C son puntos sobre una circunferencia y si la tangente en C, inter-


secta en un punto P a la prolongación de la cuerda AB, entonces P C 2 = P A · P B.

Demostración. Sabemos que un ángulo seminscrito es igual a un ángulo inscrito que sostenga
el mismo arco asi pues, el ángulo ∠P CA = ∠CBP y dado que, ∠CP A = ∠CP B los triángulos
4P CA y 4BCP son semejantes y se cumple la razón PP CA = PP B
C
y por tanto, P C 2 = P A · P B.

Ahora veremos los inversos de las proposiciones anteriores.

Proposición 3. a. Si AB, CD son dos segmentos que se intersectan en P de manera que, como
segmenteos dirigidos, P A · P B = P C · P D entonces A,B, C y D se encuentran sobre una circun-
ferencia.

b. Si A, B, C, y P son puntos tales que P, A y B están alineados y P C 2 = P A.P B, entonces


P C es tangente en C al circuncı́rculo del triángulos 4ABC.

Demostración. Denotemos por Γ la circunferencia que inscribe al triángulo ABC.

a. Supongamos que D no está sobre la circunferencia y sea D0 la intersección de P C con Γ.


Por la proposición 1 P A · P B = P C · P D0 y dado que la hipótesis dice que P A · P B = P C · P D,
tenemos que P D = P D0 , por lo que D y D0 son iguales. Contradicción. Por tanto, D debe estar
sobre la circunferencia.

b. Sea C 0 la otra intersección de P C con Γ. Por la primera proposición, P A · P B = P C · P C 0


y como por hipótesis P A · P B = P C 2 , obtenemos que P C 0 = P C y entonces C coincide con C 0 .
Podemos concliur que P C es tangente a Γ en C.

Para cualquier punto P , una circunferencia C y cualquier recta trazada por P que corte a C
en puntos A y B (A y B pueden ser iguales) que: el producto P A · P B es constante y se denomina
la potencia de punto del punto P con respecto a C.

Otra forma de calcular la potencia de un punto se comentara a continuación. Si P es exterior


a C y P C es tangente a C por P , la potencia es P C 2 . Si O es el centro de la circunferencia C y
radio r, tenemos que, por el teorema de Pitágoras: P C 2 = P O2 − r2 .

UES-FCNM-Escuela de Matemática-Licenciatura en Matemática y Licenciatura en Estadı́stica


pág. 71
Geometrı́a I. 7 Semejanza de Triángulos.

Ahora bien, si P es un punto dentro de la circunferencia y que está sobre un diámetro AB,
podemos calcular la potencia como P A · P B. Pero vemos que P A lo podemos escribir como r − P O
y P B como r + P O, o viceversa, dependiendo de si P esta mas cerca de A o de B. De ahı́ que
P A · P B = (r − P O)(r + P O) = r2 − P O2 . Si un punto esta sobre la circunferencia sabemos que
su potencia es cero, lo cual también es descrito por r2 − P O2 .

Para teminar podemos decir que la potencia de punto de P con respecto a la circunferencia
C = (O, r) es P O2 − r2 . Además, la potencia es positiva, cero o negativa, dependiendo si P se
encuentra fuera, sobre o dentro de la circunferencia.

Eje Radical
Problema. ¿Cual es el lugar geométrico de los puntos cuya potencia de punto con respecto a
dos circunferencias no concéntricas?

7.5. Problemas
a b c
1. Los puntos A, B, C y D forman una cuaterna armónica. Si AB
+ AD
= AC
; hallar a + b + c.

2. Dados los puntos A, B, C, D y E son colineales y consecutivos de modo que AB > BC


yBD > DE. Se sabe que√ AB y BD son secciones áureas de AC y BE respectivamente. Si
3− 5
BC = 2CD y AE = 2 ; calcular AC.

3. Sean AB y CD las bases del trapecio ABCD, cuyas diagonales se intersecan en E perpen-
dicularmente. Si AD = 13, AE = 12 y CE = 4 encuentre las longitudes de CD y AB.

4. En la figura 85, el 4ABC es equilátero, sus lados tienen longitud 3 y P A es paralela a BC.
Si P Q = QR = RS, encontrar la longitud de CS.

Figura 85:

5. Sea ABCD un trapecio de bases BC y AD, sus diagonales se cortan en E. Si BE = 3,


ED = 4 y CE = 2, determine la medida de AE.

6. Las bases de un trapecio miden 3 y 5, y si su altura mide 4. Encontrar la distancia desde el


punto de corte de las diagonales hasta la base mayor.

UES-FCNM-Escuela de Matemática-Licenciatura en Matemática y Licenciatura en Estadı́stica


pág. 72
Geometrı́a I. 7 Semejanza de Triángulos.

7. En la figura adjunta, el 4ABC es rectángulo en A y el 4ADB


es rectángulo en D. El punto E es el punto de intersección de
los segmentos AD y BC. Si AC = 15, AD = 16 y BD = 12,
calcule el área del 4ABE.

8. El 4ABC es rectángulo en B. Se dibuja un rectángulo BEDF con D sobre la hipotenusa,


BC 1
E y F sobre BC y AB, respectivamente. Si AB = 1, demuestre que BE = 1−DE .
9. Considérese los puntos A, B, C y D tales que A y B están sobre el segmento OC y OD respec-
tivamente, donde O es el centro de la circunferencia de radio r (Ver figura 86).
 Si OA· OC =
2
r
r2 = OB· OD, demuestre que el 4AOB ' 4DOC y que CD = AB.20
OA· OB

Figura 86:

10. Sobre la circunferencia de centro O, se trazan los diámetros AB y CD tales que AB ⊥ CD.
Sea P un punto sobre el arco CBD y Q el punto de intersección de las cuerdas AP y CD.
Si DO = 1, demuestre que AP · AQ = 2.
11. Un segmento de recta AB es divido por los puntos interiores K y L de manera que AL2 =
AK· AB. Sea P un punto exterior al segmento AB tal que AP = AL. Pruebe que ∠KP L =
∠LP B. Figura 87.

Figura 87:

12. En la figura 88, AB y AC son tangentes a la circunferencia, y CE ⊥ BD, siendo BD un


diámetro. Probar que BE ·BO = AB ·CE.
20
La medida del segmento CD se denomina Distancia Inversa.

UES-FCNM-Escuela de Matemática-Licenciatura en Matemática y Licenciatura en Estadı́stica


pág. 73
Geometrı́a I. 7 Semejanza de Triángulos.

Figura 88:

1 1 1
13. Demostrar que + = si se cumple que AX k BY k CZ. (Ver figura 89.)
AX BY CZ

Figura 89:

14. En la figura 90, el 4ABC es rectángulo. Se construyen exteriormente los cuadrados ABEF
y BCP Q. Demostrar que BM = BN .

Figura 90: .

15. Sean O, P y R los centros de las tres circunferencias. Si OR = r y Q es la intersección de


P O con la circunferencia de centro R, demuestre que OP · OQ = r2 . Ver figura 91.

UES-FCNM-Escuela de Matemática-Licenciatura en Matemática y Licenciatura en Estadı́stica


pág. 74
Geometrı́a I. 7 Semejanza de Triángulos.

Figura 91:

16. Si en un triángulo rectángulo se traza la altura correspondiente a la hipotenusa, entonces:

a) Los dos nuevos triángulos que resultan, son semejantes entre si y semejantes al triángulo
original.
21
b) La altura es media proporcional entre los segmentos que ella determina sobre la
hipotenusa.
c) Cada cateto es media proporcional entre la hipotenusa y la proyección del cateto sobre
la hipotenusa.
d ) Demuestre el teorema de Pitágoras.

17. Si dos triángulos tienen sus lados respectivamente paralelos o respectivamente perpendicu-
lares, entonces los dos triángulos son semejantes.

18. Sean ABC y A0 B 0 C 0 dos triángulos semejantes con AAB BC CA


0 B 0 = B 0 C 0 = C 0 A0 = k. Demuestre que:

la razón entre los perı́metros de los triángulos es k y que la razón entre sus áreas es k 2 .

19. Teorema de Menelao. Dado el 4ABC, sea P un punto sobre la recta AB, Q un punto
sobre la recta BC, R un punto sobre la recta CA. Si los puntos P , Q, R están alineados
AP BQ CR
entonces = 1.
P B QC RA

Figura 92: Teorema de Menelao.


21
Si b es una magnitud tal que ab = cb , entonces decimos que b es media proporcional entre a y c,o de manera
equivalente: b es media proporcional entre a y c si y solo si b2 = a· c.

UES-FCNM-Escuela de Matemática-Licenciatura en Matemática y Licenciatura en Estadı́stica


pág. 75
Geometrı́a I. 7 Semejanza de Triángulos.

Para demostrar este teorema, sea W un punto sobre la recta P QR tal que BW k AC:

a) Demuestre que los triángulos AP R y BP W son semejantes.


b) Demuestre que los triángulos CQR y BQW son semejantes.
AP BQ CR
c) De los literales a) y b) deduzca que = 1.
P B QC RA

20. Teorema de Ceva. Dado el 4ABC, sea P


un punto sobre el recta AB, Q un punto so-
bre la recta BC y R un punto sobre la recta
CA. Si las rectas AQ, CP , BR concurren,
AP BQ CR
entonces = 1.
P B QC RA

Para demostrar este teorema, sean W y V los puntos de intersección de la recta que pasa
por B paralela a AC, con las rectas CP y AQ, respectivamente.

a) Demuestre que 4AP C ∼ 4BP W y que 4AQC ∼ 4V QB.


b) Demuestre que 4BW O ∼ 4RCO y que 4BV O ∼ 4RAO.
AP BQ CR
c) Utilice los literales a) y b) para probar que = 1.
P B QC RA
21. Sea ABC un triángulo, con D sobre AB, E sobre AC y F sobre BC, tal que DE es paralelo
a BC, Demuestre que BF = F C

22. Sea ABC un triángulo, con D sobre AB, con E sobre BC y F sobre AC, tal que AD = 2BD
y F A = 2CF . Demuestre que E es punto medio de B.

23. Sea Γ una circunferencia y dado un triángulo ABC, sean L, L0 , M, M 0 , N, N 0 los puntos de
corte de la circunferencia con el triángulo , sobre los lados BC, AC, AB respectivamente.
Demostrar que si AL, BM, CN concurren, entonces AL0 , BM 0 , CN 0 concurren.

24. En el triángulo ABC, rectángulo en A , se consideran las circunferencias inscritas y circuns-


critas. La recta AM es tangente a la circunferencia circunscrita en el punto A (M es punto
de BC). S y R son los puntos de tangencia de la circunferencia inscrita con los catetos AC y
AB, respectivamente. La recta RS corta a la recta BC en N . Las rectas AM y SR se cortan
en U . Demostrar que el triángulo U M N es isósceles.

UES-FCNM-Escuela de Matemática-Licenciatura en Matemática y Licenciatura en Estadı́stica


pág. 76
Geometrı́a I. 7 Semejanza de Triángulos.

25. Demuestre que si ABC es un triángulo y AA0 es su bisectriz externa (con A0 sobre BC)
0
entonces BA
A0 C
= AB
AC

26. Demuestre que si ABC es un triángulo y suponga que las bisectrices internas de B y C
cortan a CA y AB en b0 y C 0 respectivamente y que la bisectriz externa de A corta a BC en
A0 . Demuestre que A0 , B 0 , C 0 son colineales.

27. Si dos cuerdas se interceptan en el interior de una circunferencia entonces el producto de las
medidas de los segmentos determinados por el punto de intersección en una de las cuerdas
es igual al producto de las medidas de los segmentos determinados en la otra cuerda.

28. Si dos segmentos se interceptan en un punto que esta en el interior de los dos segmentos
y el producto de las medidas de los segmentos determinados por el punto de intersección
en el primer segmento es igual al producto de las medidas de los segmentos determinados
por el punto en el segundo segmento,entonces los extremos de los segmentos están sobre una
circunferencia.

29. Si desde un punto P exterior a una circunferencia se trazan dos semirrectas secantes que
cortan a la circunferencia en los puntos A, B y C, D respectivamente, entonces P A· P B =
P C· P D.

30. Si desde un punto P se trazan dos semirrectas con los puntos A, B sobre una y los puntos C,
D sobre la otra, tales que P A· P B = P C· P D, entonces los puntos A, B, C, D están sobre
una circunferencia.

31. Si desde un punto exterior a una circunferencia se trazan dos semirrectas, una tangente y la
otra secante, entonces el segmento entre el punto y el punto de tangencia es media propor-
cional entre los segmentos determinados entre el punto exterior y los puntos de intersección
de la secante con la circunferencia. 22

32. Si P es un punto sobre el mismo plano que una circunferencia de centro O y radio r, y d es
la distancia del punto P al centro O de la circunferencia, demuestre que:

a) Si P está en el interior de la circunferencia, entonces la potencia de P es r2 − d2 .


b) Si P está en el exterior de la circunferencia, entonces la potencia de P es d2 − r2 .
c) Si P está sobre de la circunferencia, entonces la potencia de P es cero.

33. En un triángulo ABC los puntos D y E están en los lados AC y AB de forma tal que los
ángulos ∠ADE = ∠ABD. Si AE = 2 y BE = 3 encuentre AD.

34. El cuadrilátero ABCD es cı́clico y la intersección de los lados AC y BD es P . Halle BP si


P C = 4, DP = 2 y AP = 8.

22
Los problemas anteriores nos permite establecer la siguiente definición de Potencia de un punto con respecto
a una circunferencia: La potencia de un punto P con respecto a una circunferencia de centro O y radio r es el
producto P A· P B, donde A y B son los puntos de intersección de la circunferencia con una recta que pasa por P .

UES-FCNM-Escuela de Matemática-Licenciatura en Matemática y Licenciatura en Estadı́stica


pág. 77
Geometrı́a I. 7 Semejanza de Triángulos.

35. En un cuadrilátero ABCD los ángulos en ∠A y en ∠C son rectos. Si los lados AB = 3,


AD = 4 y DP = 2, hallar hallar AP · P C.

36. Por un punto P sobre la cuerda común AB de dos circunferencias que se intersectan se tra-
zan las cuerdas KM sobre el primer cı́rculo y LN sobre el segundo cı́rculo. Pruebe que el
cuadrilátero KLM N es concı́clico.

37. En el triángulo ABC con circuncentro O, las alturas BE y CF se cortan en H. Las rectas
OB y HC se cortan en P . Si HP = 3, OP = 2 y P C = 5, ¿cuánto vale OB?

38. La recta OA es tangente a una circunferencia en el punto A y la cuerda BC es paralela a


OA. Las rectas OB y OC intersectan a la circunferencia por segunda vez en los puntos K y
L, respectivamente. Pruebe que la recta KL divide al segmento OA por la mitad.

39. Dada una circunferencia S, los puntos A y B sobre esta y C en la cuerda AB. Para cualquier
circunfenrencia S 0 tangente a la cuerda AB en el punto C y que intersecta a S en P y Q,
considere el punto de intersección M de las rectas AB y P Q. Pruebe que la posición de M
no depende de la eleccción de S 0

40. Desde A se trazan una recta tangente a la circunferencia S en C y una recta que corta a S
en los puntos ByD. Sea E otro punto en la circunferencia tal que CE es tangente a AD. Si
CA = 6, AB = 12 y CE = 4BD hallar CE y BD. Si P es el punto de intersección de CD
con BE cual es la potencia de punto de P con respecto a S.

41. Dos circunferencias se intersectan en los puntos A y B y sea M N su tangente comun. Pruebe
que la recta AB divide a M N por la mitad.

42. Pruebe que el eje radical de dos circunferencias es el lugar geométrico de los puntos P tales
que al trazar las tangentes a las circunferencias la distancia de P a ambos puntos de tangen-
cia es la misma.

43. A partir de un punto P exterior a una circunferencia de centro O se trazan las tangentes

P Q y la secante P BA, tal que P Q = AB = 2, si el radio de la circunferencia mide 1 + 5 ,
hallar ∠BOP .

44. Dado el 4ABC se construye un cuadrado P QRS con P en AB, Q en AC, R y S en BC.
Sea H el pie de la altura desde A hacia BC. Demuestre que:
1 1 1
a) = +
PQ AH BC
b) (ABC) = 2(P QRS) si y sólo si AH = BC.

UES-FCNM-Escuela de Matemática-Licenciatura en Matemática y Licenciatura en Estadı́stica


pág. 78
Geometrı́a I. 7 Semejanza de Triángulos.

45. Sea P un punto en el interior del 4ABC. Se trazan por P las paralelas a los lados del
triángulo, que queda dividido en tres triángulos y tres paralelogramos. Si las áreas de los tres
triángulos de la subdivisión son, en algún orden, 9, 16 y 25, hallar el área del 4ABC.

46. Las tres circunferencias de la figura 93 tienen el mismo radio r, sus centros son colineales y
la circunferencia de centro O2 es tangente a las otras dos. Por A se traza una tangente a la
circunferencia de centro O3 . Obtenga el valor del segmento BC en función de r.

Figura 93:

47. Sea ABCD un rombo, con AC = 6 y BD = 8. Se construyen exteriormente los cuadrados


ADEF y CDHG, cuyos centros son O1 y O2 , respectivamente (Vea figura 94). Calcular la
medida del segmento O1 O2 .

Figura 94:

48. Sea ABCD un cuadrado con P y Q sobre AB y BC tales que BP = BQ. Sea H el pie de
la perpendicular de B a P C. Demuestre que DHQ = 90°.
49. Alrededor de una circunferencia se construyen diez circunferencias tangentes a la original
y tangentes entre sı́ (Véase figura 95). Demuestre que la suma de las áreas de las diez
circunferencias es el doble del área de la circunferencia mayor.

UES-FCNM-Escuela de Matemática-Licenciatura en Matemática y Licenciatura en Estadı́stica


pág. 79
Geometrı́a I. 7 Semejanza de Triángulos.

Figura 95:

50. En un 4ABC el ∠CAB = 120. Encuentre la medida de la bisectriz interna del ∠CAB en
función de los lados adyacentes.

51. El 4ABC tiene lados de 13, 14 y 15 unidades. El 4A0 B 0 C 0 está dentro del 4ABC con
lados paralelos a los de éste y a 2 unidades de distancia de los lados del mismo. Calcule
(ABC) − (A0 B 0 C 0 ).

UES-FCNM-Escuela de Matemática-Licenciatura en Matemática y Licenciatura en Estadı́stica


pág. 80
Geometrı́a I. 7 Semejanza de Triángulos.

7.6. Puntos y Rectas Notables del Triángulo.


7.6.1. Medianas
Definición: En un triángulo, una mediana es el segmento de recta que une un vértice con el
punto medio del lado opuesto.

Teorema: Las tres medianas de un triángulo concurren en un punto llamado el Centroide 23


del triángulo y usualmente es denotado por G. Además, las medianas de cortan mutuamente en
razón 2:1.

Demostración: Dado el 4ABC sean A0 , B 0 , C 0 , los puntos medios de BC, CA, AB, res-
pectivamente. Defina G como la intersección de BB 0 con CC 0 . Por el teorema de la base media,
B 0 C 0 k BC y 2B 0 C 0 = BC; observe que 4BCG ' B 0 C 0 G, con razón de semejanza 2, por lo que

GB GC
0
= =2
GB GC 0
Análogamente, si G∗ = AA0 ∩ BB 0 se cumple
G∗ B G∗ A
= ∗ 0 =2
G∗ B 0 GA
Ası́, G y G∗ dividen al segmento BB 0 en dos segmentos cuya razón es 2:1, por lo que G = G∗ , lo
cual implica que AA0 , BB 0 , CC 0 concurren y
GA GB GC
0
= 0
= =2
GA GB GC 0

7.6.2. Mediatrices
Definición: Dado un segmento AB, la mediatriz del segmento es el lugar geométrico de puntos
que equidistan de A y B, i.e., un punto P está sobre la mediatriz de AB si y sólo si P A = P B.

Teorema: La mediatriz de AB es una recta l perpendicular a AB y que pasa por su punto


medio.

Demostración: Sea M el punto medio de AB, y l pasa por M y l ⊥ AB. En primer lugar
se probará que todos los puntos de l satisfacen la definición de mediatriz: Por definición de punto
medio M A = M B. por lo que claramente M pertenece a la mediatriz de AB; sea P un punto
de l distinto de M , por criterio LAL, 4P M A ≡ 4P M B por lo que P A = P B. Ahora, cabe
preguntarse si existe algún punto fuera de l que también cumpla la definición: suponga P 0 tal que
P 0 A = P 0 B, esto implica que 4P 0 AB es isósceles, y entonces ∠P 0 AB = ∠P 0 BA; si M 0 es la pro-
yección de P 0 sobre AB, por criterio ALA 4P 0 AM 0 ≡ 4P 0 BM 0 , lo cual implica que M 0 A = M 0 B,
es decir que M 0 = M , y esto obliga a que P 0 esté sobre l (ya que P 0 M 0 = l).

Teorema: Las mediatrices de un 4ABC concurren en un punto que equidista de los vértices
del triángulo, llamado el Circuncentro del 4ABC
23
También conocido como Geocentro, Centro de Gravedad, Baricentro, o más formalmente Equibaricentro.

UES-FCNM-Escuela de Matemática-Licenciatura en Matemática y Licenciatura en Estadı́stica


pág. 81
Geometrı́a I. 7 Semejanza de Triángulos.

Figura 96: Concurrencia de Mediatrices, Circuncentro y Circuncı́rculo.

Usualmente, el circuncentro es denotado por O, y R representa la distancia del circuncentro a


los vértices
R = OA = OB = OC
A esta distancia se le llama Circunradio del 4ABC. Ası́, O es el centro de una circunferencia que
pasa por A, B, C, cuyo radio es R, llamada el Circuncı́rculo del 4ABC.24

Demostración: Sea O la intersección de las mediatrices de AB y BC, por el teorema anterior,


como O pertenece a la mediatriz de AB se cumple OA = OB, y como también pertenece a la
mediatriz de BC, OB = OC; entonces OC = OA, y utilizando de nuevo el teorema anterior,
O debe pertenecer a la mediatriz de CA. Ası́, las tres mediatrices concurren en O, y este punto
equidista de los vértices del 4ABC.

Corolario: Dado un triángulo, existe una circunferencia que pasa por los tres vértices (el cir-
cuncı́rculo); además, esta circunferencia es única.

Una observación importante es que la mediatriz del lado de un triángulo NO siempre pasa por
el vértice opuesto; de hecho, esto sólo se da si el triángulo es isósceles.

7.6.3. Alturas
La altura es un concepto que está intrı́nsecamente relacionado con la distancia de un punto a
una recta; la altura es la recta que debe trazarse para determinar esta distancia, i.e., es una recta
que pasa por el punto y es perpendicular a la recta. A la intersección entre la altura y la recta
generalmente se le llama pie de la altura, o también (más formal) proyección del punto sobre la
recta. En particular, para triángulos, definiremos la altura de la siguiente forma:

Definición: Dado un triángulo, una altura es una recta que pasa por un vértice y es perpen-
dicular al lado opuesto.

Es importante observar que el pie de la altura NO siempre pertenece a un lado; de hecho,


una altura puede estar “al interior” de un triángulo, coincidir con un lado, o estar completamente
24
En ocasiones, denotaremos a esta circunferencia por Γ(ABC).

UES-FCNM-Escuela de Matemática-Licenciatura en Matemática y Licenciatura en Estadı́stica


pág. 82
Geometrı́a I. 7 Semejanza de Triángulos.

afuera de un triángulo.

Teorema: Las alturas de un triángulo concurren en un punto, llamado el Ortocentro del


triángulo, usualmente denotado por H.25

Demostración: Dado el 4ABC, se construyen los puntos A1 , B1 , C1 , tales que ABA1 C,


BCB1 A, CAC1 B son paralelogramos. Observe que el 4ABC es el triángulo medial del 4A1 B1 C1 ,
y que las alturas del 4ABC son las mediatrices del 4A1 B1 C1 ; como las mediatrices de un triángu-
lo concurren (en este caso, las del 4A1 B1 C1 ), las alturas del 4ABC concurren.

La altura también puede escribirse en términos de lugar geométrico:

Teorema: La recta l es perpendicular a AB si y sólo si AL2 − LB 2 es constante. Es decir, que


una recta perpendicular a AB es el lugar geométrico de los puntos L que satisfacen la condición
anterior.

Demostración: sea P la intersección de l con AB, y L un punto arbitrario sobre l; por Pitágo-
ras se tiene AL2 − LB 2 = AP 2 − P B 2 , y el término derecho de la igualdad es constante. La otra
dirección de la implicación se prueba por contradicción.

De esa definición también puede fabricarsele una demostración del teorema anterior, sin em-
bargo, no se aborda porque la prueba se basa en un resultado sofisticado llamadado Teorema de
Steiner.26

7.6.4. Bisectrices
Definición: La bisectriz de un ángulo es una recta que “divide” al ángulo en dos ángulos de
igual magnitud.

Teorema: El lugar geométrico de puntos que equidistan de dos rectas dadas, generan un par
de rectas perpendiculares llamadas bisectriz interna y bisectriz externa del ángulo formado por las
rectas.

Demostración: Suponga que las rectas se cortan en un punto O; sean a, b las rectas dadas,
y P un punto que equidista de ellas; si A y B son las proyecciones de P sobre a y b, respecti-
vamente, entonces P A = P B. Observe que por criterio LLL (utilizando Pitágoras previamente),
4OAP ≡ 4OBP , por lo que ∠P OA = ∠P OB, i.e., P pertenece a la bisectriz del ∠AOB. Clara-
mente aquı́ se dan dos casos, recuerde que para definir el ángulo entre a y b se utilizan únicamente
semi-rectas, por lo que las rectas a y b definen cuatro ángulos, que por parejas pueden ser opuestos
por el vértice o suplementarios; de estos se escoge cualquiera de ellos como referencia, entonces, si
∠AOB coincide con éste o con el opuesto por el vértice, la recta P O es llamada bisectriz interna, y
en caso contrario, bisectriz externa. Ası́, el lugar geométrico son dos rectas, y su perpendicularidad
se basa en los pares de ángulos que son suplementarios. Finalmente, si a k b, el lugar geométrico
25
El triángulo formado por los pies de las alturas de un 4ABC es llamado el triángulo órtico del 4ABC.
26
Sean l, m, n, tres rectas perpendiculares a los lados del AB, BC, CA del 4ABC, respectivamente. Sean
L, M , N , puntos arbitrarios sobre l, m, n, respectivamente. Entonces las rectas l, m, n concurren si y sólo si
AL2 + BM 2 + CN 2 = N A2 + LB 2 + M C 2 .

UES-FCNM-Escuela de Matemática-Licenciatura en Matemática y Licenciatura en Estadı́stica


pág. 83
Geometrı́a I. 7 Semejanza de Triángulos.

es una recta paralela a a y b que se ubica entre ellas a igual distancia de ambas (este es un caso
extraño de bisectriz interna, sin embargo, en ocasiones es útil tener esta convención en mente; peor
aún, la bisectriz externa es una recta ideal llamada recta al infinito).

Teorema: Las bisectrices internas de un 4ABC concurren en un punto, llamado el Incentro


del 4ABC, usualmente denotado por I. La distancia de I a los tres lados del triángulo es igual a
un número r, llamado el Inradio del 4ABC, y de aquı́ que la circunferencia de centro I y radio r
sea tangente a los lados del triángulo; dicha circunferencia es llamada el Incı́rculo del 4ABC.27

Figura 97: Concurrencia de Bisectrices Internas, Incentro e Incı́rculo.

Demostración Sea I la intersección de las bisectrices internas de ∠A y ∠B (obviamente, I


está en el interior del 4ABC); como I pertenece a la bisectriz interna del ∠A, por el teorema
anterior dist(I, AB) = dist(I, AC), y análogamente, como I pertenece a la bisectriz interna del
∠B, dist(I, AB) = dist(I, CB); entonces dist(I, AC) = dist(I, CB), y de nuevo por el teorema
anterior y dado que I está al interior del triángulo, I pertenece a la bisectriz interna del ∠C. Ası́,
las tres bisectrices internas concurren en un punto que equidista de los lados del triángulo.

Es importante notar que las intersección de una bisectrices interna con el lado opueto del
triángulo NO siempre coincide con el puntos de tangencia del incı́rculo;28 de hecho, esto ocurre
sólamente si el triángulo es isósceles.

Corolario: Dado un triángulo, existe una circunferencia que es tangente interiormente a los
tres lados (el incı́rculo); además, esta circunferencia es única.29

7.6.5. Problemas
1. Sea I el incentro del triángulo ABC. Si el ∠AIB = 130◦ , determine la medida del ∠ACB y
del ∠ACI.

2. Las áreas de los seis triángulos AGB 0 , AGC 0 , BGA0 , BGC 0 , CGA0 , CGB 0 son iguales e
iguales a un 61 del área del triángulo ABC.
27
En algunas ocasiones denotaremos al incı́rculo por Λ(ABC).
28
En la figura, el 4ABC es llamado triángulo tangencial del 4DEF .
29
Existen 3 circunferencias más que son tangentes a los tres lados del triángulo, llamados excı́rculos; estas cir-
cunferencias se ubican en el exterior del triángulo.

UES-FCNM-Escuela de Matemática-Licenciatura en Matemática y Licenciatura en Estadı́stica


pág. 84
Geometrı́a I. 7 Semejanza de Triángulos.

Figura 98:

3. Los cuatro triángulos AB 0 C 0 , BC 0 A0 , CA0 B 0 , A0 B 0 C 0 ,30 son congruentes entre si y semejantes


al 4ABC con razón de semejanza 12 .

4. El centroide del 4ABC coincide con el centroide del triángulo medial 4A0 B 0 C 0 . Además,
estos dos triángulos tienen lados correspondientes paralelos (triángulos homotéticos).

5. En la figura 99, G es el centroide. Si GD = 2 y el área sombreada vale 5, calcule AD y el


(ABC).

Figura 99:

6. Demostrar que las paralelas a los lados de un 4ABC, trazadas por el centroide G dividen
cada lado en tres partes iguales.

7. ABCD es un paralelogramo de centroide (baricentro) E, M es el punto medio de AD, y F


es la intersección de AC con BM . Si el área de ABCD es 1, calcule el área del cuadrilátero
DEF M .

8. En el 4ABC, se traza la mediana AM . Demostrar que si BM = AM , entonces el triángulo


es rectángulo en A.

9. La suma de las distancias del centroide a los puntos medios de los lados de un triángulo es
20. Calcule la suma de las medianas del triángulo.

10. La mediana tiene longitud menor que la semisuma de los lados adyacentes, es decir AA0 <
b+c c+a a+b
, BB 0 < , CC 0 < .
2 2 2
30
El 4A0 B 0 C 0 es llamado el triángulo medial del 4ABC.

UES-FCNM-Escuela de Matemática-Licenciatura en Matemática y Licenciatura en Estadı́stica


pág. 85
Geometrı́a I. 7 Semejanza de Triángulos.

11. Dado el 4ABC, sean D y E puntos variables sobre los lados AB y AC respectivamente tales
que BC k DE. Entonces, la mediana AA0 puede definirse como el lugar geométrico de los
puntos P tales que P ∈ CD ∩ BE.31

12. Siempre es posible construir un triángulo XY Z con las medianas AA0 , BB 0 , CC 0 de un


4ABC dado. Además, los segmentos que unen el centroide del 4XY Z con sus vértices son
iguales a la mitad de los lados del 4ABC.

13. En el 4ABC, AB = BC y la mediatriz de BC interseca a la mediana BM en L. Si ∠LCB =


25, determine la medida del ∠LAC.

14. Ley del Seno. Dado un 4ABC, se cumple que


sen∠A sen∠B sen∠C 1
= = =
a b c 2R

15. Las reflexiones de H con respecto a los lados del 4ABC caen sobre el circuncı́rculo del
mismo, es decir HHa = Ha X y análogo para los otros lados.

Figura 100:

16. Las reflexiones de H con respecto a los puntos medios de los lados del triángulo, caen sobre
el circuncı́rculo del mismo.

17. Si O y H son el circuncentro y el ortocentro de un 4ABC, respectivamente, entonces


∠BAH = ∠CAO.

18. La altura AHa es bisectriz del ∠Hb Ha Hc .

19. Los circuncı́rculos de 4ABC, 4ABH, 4BCH, 4CAH tienen igual radio.
31
Si D = A se define P = A, y cuando D = B entonces P es punto medio de BC.

UES-FCNM-Escuela de Matemática-Licenciatura en Matemática y Licenciatura en Estadı́stica


pág. 86
Geometrı́a I. 7 Semejanza de Triángulos.

20. La perpendicular trazada desde A al lado Hb Hc del triángulo órtico, pasa por el circuncentro
del 4ABC.

21. A, B, C y H forman un cuadrilátero ortocéntrico, es decir que cada punto es el ortocentro


del triángulo formado por los otros tres.

22. El ortocentro de un triángulo está al interior, sobre un vértice, o afuera del triángulo, si el
triángulo es acutángulo, rectángulo, u obtusángulo, respectivamente.

23. El circuncentro del 4ABC es el ortocentro del triángulo medial 4A0 B 0 C 0 .

24. Sea O el circuncentro del 4ABC. Si ∠AOC = 100 y ∠OCB = 30, determine la medida de
los ángulos del 4ABC.

25. Hallar los ángulos de un triángulo cuyo triángulo órtico tiene ángulos de 20, 50 y 110.

26. Sea ABC un triángulo obtusángulo de circuncentro O y altura AD. Si ∠OAB = 25 y


∠OCB = 15, calcule el ∠DAB.

27. El 4ABC de circuncentro O y altura BD. Si ∠DAB = 35 y ∠OBD = 10 encontrar los


ángulos del triángulo ABC.

28. En la figura 101, AB es diámetro de la circunferencia. Si X es la intersección de CG con


AB, calcular el ∠CXB.

Figura 101:

29. En el 4ABC, se trazan la altura AH y la mediana BM . Demuestre que el 4M HC es


isósceles.
AB
30. Un 4ABC es rectángulo en C, ∠A = 75 y CH es altura. Demuestre que CH = .
4
31. Sea O el circuncentro del 4ABC con ∠C = 45 y sea D el pie de la altura desde A. Calcule
la medida del ∠ODC.

32. Dado el 4ABC isósceles con ∠A = 90, sean P y Q son puntos dentro del triángulo tales que
BP = AQ y AP = CQ. Si BP y CQ se cortan en R, demostrar que AR ⊥ P Q.

UES-FCNM-Escuela de Matemática-Licenciatura en Matemática y Licenciatura en Estadı́stica


pág. 87
Geometrı́a I. 7 Semejanza de Triángulos.

33. Se ubican los puntos M y K sobre los lados BC y CD del cuadrado ABCD, respectivamente,
de modo que M C = KD. Sea P la intersección de M D y BK, demuestre que AP ⊥ M K.
34. Sean D, E, F los puntos de tangencia del incı́rculo sobre los lados BC, CA, AB del 4ABC.
Demuestre que se cumplen las siguientes relaciones, donde s denota el semiperı́metro del
triángulo:
AE = AF = s − a
BD = BF = s − b
CD = CE = s − c
35. El ortocentro del 4ABC es el incentro de su triángulo órtico.
36. Dado un 4ABC, su triángulo órtico y su triángulo tangencial tienen lados correspondientes
paralelos (triángulos homotéticos).
37. Las bisectrices exteriores de ∠B y ∠C, junto con la bisectriz interior de ∠A, concurren en
un punto, llamado el Excentro con respecto al vértice A (Véase figura 102), usualmente
denotado por Ia . Este punto es equidistante a los lados del 4ABC, dicha distancia es el
Exradio respecto a A, usualmente denotado por ra . Ası́, la circunferencia de centro Ia y radio
ra es tangente exteriormente a los lados del 4ABC, y es llamada el Excı́rculo respecto a
A.32

Figura 102:

38. I es ortocentro del 4Ia Ib Ic (Véase figura 102). Además se cumple:


AX = AZ = s
BX = BY = s − b
CY = CZ = s − c
32
Análogamente se definene los excı́rculos con respecto a los otros vértices.

UES-FCNM-Escuela de Matemática-Licenciatura en Matemática y Licenciatura en Estadı́stica


pág. 88
Geometrı́a I. 7 Semejanza de Triángulos.

39. En un 4ABC, la bisectriz exterior del ∠ABC y la bisectriz exterior del ∠BCA se cortan en
D. La paralela a BC por D corta a AC en L y a AB en M . Si LC = 5 y M B = 7, hallar
LM .

40. El 4ABC es rectángulo en A. Si I es el incentro, calcular ∠BIC.

41. En un 4ABC, el ∠ABC − ∠CAB = 90. Sean D y E los pies de las bisectrices interior y
exterior del ∠BCA respectivamente. Demuestre que CD = CE.

42. En el 4ABC, AB < AC, AD es bisectriz, y E es un punto en AB tal que el ∠EDB = 90.
El punto F sobre AC es tal que el ∠BED = ∠DEF . Demuestre que el ∠BAD = ∠F DC.

43. En el 4ABC se trazan las bisectrices interiores BD y CE tales que D es el punto sobre AC,
E es el punto sobre AB, 2∠BDE = 3∠B y ∠CED = 2∠B. Calcular los ángulos del 4ABC.

44. Dado el 4ABC con ∠A = 90, sea D el pie de la perpendicular desde A. Sean además I y
J los incentros respectivos de 4ABD y 4ACD. Demostrar que la bisectriz del ∠BAC es
perpendicular a IJ.

45. Un triángulo es isósceles si cumple alguna de las siguientes condiciones:

a) Dos medianas son iguales.


b) Dos alturas son iguales.
c) Dos bisectrices son iguales.33

46. Teorema de la Bisectriz: Dado el 4ABC, sean P y P 0 sobre BC. Se cumple que AP y
AP 0 son la bisectriz interna y la bisectriz externa del ∠A si y sólo si
BP BA BP 0
= = 0
PC AC PC
Sugerencia: Para demostrar la primera igualdad, trace CD k AP con E sobre la prolongación
de AB.

47. (*) De acuerdo con los datos de la gráfica 103, calcular el valor de AB.

Figura 103:

33
Este caso es aparentemente tan sencillo como los anteriores, pero realmente es un resultado muy complicado y
recibe el nombre de Teorema de Steiner-Lehmus.

UES-FCNM-Escuela de Matemática-Licenciatura en Matemática y Licenciatura en Estadı́stica


pág. 89
Geometrı́a I. 7 Semejanza de Triángulos.

48. Dos circunferencias son tangentes internamente en P , y una cuerda AB de la circunferencia


de radio mayor es tangente en Q a la otra circunferencia. Ver figura 104.

a) Demuestre que P Q es bisectriz del ∠AP B.


b) Llame A0 y B 0 a las otras intersecciones de P A y P B con la circunferencia de radio menor
y suponga que AB = 15, P A0 = 3 y P B 0 = 2; calcule AQ y BQ.

Figura 104:

seguirá siendo válida. Si hacemos ` coincida con alguna de las alturas, las reflexiones clara-
mente concurren en uno de los vértices del triángulo; por tanto también serán concurrentes
para cualquier otra posición de `.

49. (*) Teorema de Poncelet: Demuestre si 4ABC es un triángulo rectángulo con ∠A = 90°,
entonces 2(r + R) = b + c.

50. Demuestre que las mediatrices de un cuadrilátero son concurrentes si y sólo si es cı́clico.

51. Las bisectrices BP y CQ del 4ABC se cortan en I. Demuestre que si ∠BAC = 60◦ entonces
4P QI es isósceles.

52. Demuestre que el cuadrilátero convexo ABCD es inscribible si y sólo si los incı́rculos respec-
tivos del 4ABC y 4CDA son tangentes.

53. Demuestre que las bisectrices internas de un cuadrilátero son concurrentes si y sólo si es
inscribible.

54. Demuestre que todo rombo es inscribible.

55. (*) Sea ABCD un paralelogramo. Q es el punto medio de AD, F el pie de la perpendicular
por B sobre QC. Probar que AF = AB.

56. Dado el rombo ABCD, se trazan las bisectrices internas de ∠DAC, ∠CAB, ∠BCA, ∠ACD,
y cortan a DC, CB, BA, AD en P , Q, R, S, respectivamente. Demuestre que P QRS es un
rectángulo.

57. (*) Sea ABCD un cuadrilátero tal que AB = CD. Las mediatrices de AC y BD se cortan
en P . Probar que ∠P AC = ∠P CA = ∠P BD = ∠P DB.

UES-FCNM-Escuela de Matemática-Licenciatura en Matemática y Licenciatura en Estadı́stica


pág. 90
Geometrı́a I. 7 Semejanza de Triángulos.

58. (*) ABC es un triángulo y P un punto en su interior. Sean A0 , B 0 y C 0 las reflexiones de P so-
bre BC, CA y AB, respectivamente. D, E y F son los pies de las perpendiculares respectivos
desde A, B y C hacia B 0 C 0 , C 0 A0 y A0 B 0 . Probar que AD, BE y CF son concurrentes.

59. (*) (Arnoldo Aguilar) En la figura 105, ABGH, BCF G y CDEF son cuadrados. Si I es el
centro de ABGH y J = DH ∩ BG, demuestre que I, J y F están alineados.

Figura 105:

60. (*) (Arnoldo Aguilar) Sea ABC un triángulo equilátero. M y N son los puntos medios de
AB y BC, respectivamente. Exteriormente al 4ABC se construye un triángulo rectángulo
isósceles 4AP C, con ∠AP C = 90◦ . Si I es la intersección de AN y M P , demuestre que CI
es la bisectriz de ∠ACM .

61. (*) En la figura adjunta, el 4ABC es tal que ∠A = 90° y


∠B = 60°. ¿Cuál es el radio de la circunferencia?

62. (*) Dado el paralelogramo ABCD, sea M el punto medio de AB, y N la intersección de CD
con la bisectriz interna del ∠ABC. Demuestre que M C ⊥ BN si y sólo si AN es bisectriz
del ∠DAB.

63. (*) En el 4ABC, se sabe que los vértices B, C, el circuncentro O y el ortocentro H del
4ABC están todos sobre una misma circunferencia.

a) Calcule el valor del ∠A.


b) Demuestre que el incentro también pertenece al circuncı́rculo de BCOH.

64. (*) Sea ABC un triángulo de ortocentro H. Sean P y Q los pies de las perpendiculares desde
H a las bisectrices interior y exterior de A, respectivamente. Si M es el punto medio de BC,
mostrar que P , Q y M están alineados.

65. (*) En un triángulo ABC, sea M el punto medio de BC. Si se cumple que AB 6= AC y
además ∠M AC + ∠ABC = 90◦ , hallar ∠BAC.

UES-FCNM-Escuela de Matemática-Licenciatura en Matemática y Licenciatura en Estadı́stica


pág. 91
Geometrı́a I. 7 Semejanza de Triángulos.

66. (*) Sea ABC un triángulo y U un punto de su circuncı́rculo tal que AU es bisectriz. Las
mediatrices en AB y AC cortan a AU en X y Y . Sea T la intersección de BX con CY .
Demostrar que AU = T B + T C.

67. (*) (The 59th Romanian Mathematical Olympiad Final Round) Sea ABCD un rectángulo
de centro O con AB 6= BC. La perpendicular en O a BD corta a las lı́neas AB y BC en los
puntos E y F , respectivamente. Sean M y N los puntos medios de los segmentos CD y DA,
respectivamente. Probar que las lı́neas rectas F M ⊥ EN .

68. (*) Sea ABC un triángulo rectángulo, con A = 90◦ . Sea D un punto en su interior tal que
∠DAC = ∠DCA = ∠DBC = α, y AC = BD. Determine el valor de α.

69. (*) Sea ABC un triángulo y M un punto tal que ∠M AB = 10, ∠M BA = 20, ∠M AC = 40
y ∠M CA = 30. Probar que el 4ABC es isósceles.

70. (*) En la figura 106, ABCD y P QRS son cuadrados, 4ABP ≡ 4BCQ ≡ 4CDR ≡ 4DAS
y los los radios de las cinco circunferencias son iguales a r. Si a es el lado del cuadrado ABCD,
determine r en función de a.

Figura 106:

71. (*) Recta de Euler. El centroide G, el ortocentro H y el circuncentro O de un triángulo


están alineados, y además GH = 2GO.

72. Circunferencia de los 9 puntos:34 Dado un 4ABC de ortocentro H, se cumple que los
puntos medios de los lados, los pies de las alturas, y los puntos medios de HA, HB, HC, se
ubican sobre una misma circunferencia. Además, el centro de esta circunferencia es el punto
medio de HO y su radio es R2 , donde O y R son el circuncentro y el circunradio del triángulo.
Para demostrar este resultado se sugiere seguir los siguientes pasos:

a) Si Ha es el pie de la altura trazada desde A, demuestre que la reflexión de H con respecto


a Ha pertenece a circuncı́rculo del 4ABC. Resultados similares se cumplen para Hb y
Hc .
b) Si A0 es el punto medio de BC, demuestre que la reflexión de H con respecto a A0 pertenece
al circuncı́rculo del 4ABC.
34
También conocida como Circunferencia de Feuerbach.

UES-FCNM-Escuela de Matemática-Licenciatura en Matemática y Licenciatura en Estadı́stica


pág. 92
Geometrı́a I. 7 Semejanza de Triángulos.

c) De los resultados anteriores, observe que hay 9 puntos sobre el circuncı́rculo del 4ABC:
los vértices, las reflexiones de H con respecto a los pies de las alturas, y las reflexiones de
H con respecto a los puntos medios de los lados; a partir de esto, concluya que los puntos
medios de los segmentos que van de H a estos 9 puntos, también deben pertenecer en una
misma circunferencia.
d) Concluya además que el centro de esta nueva circunferencia es el punto medio de HO.
Otro camino de solución es el siguiente:
a) Sea 4A0 B 0 C 0 el triángulo medial del 4ABC. Pruebe que ∠A0 B 0 C 0 = ∠BHa C 0 y concluya
que Ha A0 B 0 C 0 es un cuadrilátero cı́clico; los mismo debe cumplirse para Hb y Hc .
b) Sea X el punto medio de HA. Demuestre que ∠B 0 A0 C 0 + ∠B 0 XC 0 = 180 y concluya que
XC 0 B 0 A0 es un cuadrilátero cı́clico; lo mismo debe cumplirse para los puntos medios de
HB y HC.
c) De lo anterior, concluya que los pies de las alturas y los puntos medios de los segmentos que
van desde H hasta los vértices del 4ABC, se ubican sobre el circuncı́rculo del 4A0 B 0 C 0 .
d) Si N es el circuncentro del 4A0 B 0 C 0 , demuestre que N , O, G forman la recta de Euler
del 4A0 B 0 C 0 y utilice sus propiedades para probar que N es el punto medio de HO.
73. El área del 4ABC, denotada por [ABC], cumple:
base × altura
[ABC] =
2
ab sen∠C bc sen∠A ca sen∠B abc
[ABC] = = = =
2 2 2 4R
[ABC] = sr
p
[ABC] = s(s − a)(s − b)(s − c). (Fórmula de Herón).

74. El circunradio, el inradio y los exradios de un triángulo cumplen:


4R = ra + rb + rc − r
[ABC] = ra (s − a) = rb (s − b) = rc (s − c)
r
(s − a)(s − b)(s − c)
r =
r s
s(s − b)(s − c)
ra =
s−a
r
s(s − a)(s − c)
rb =
s−b
s
s(s − a)(s − b)
rc =
(s − c)

75. Dado el 4ABC, sea I el incentro e Ia el excentro respecto a A.


a) Demuestre que BICIa es un cuadrilátero cı́clico.
b) Si M es la intersección de IIa con el circuncı́rculo del triángulo (M 6= A), demuestre que
dicho punto es el circuncentro de BICIa .

UES-FCNM-Escuela de Matemática-Licenciatura en Matemática y Licenciatura en Estadı́stica


pág. 93
Geometrı́a I. 7 Semejanza de Triángulos.

c) Sea M 0 el punto diametralmente opuesto a M en el circuncı́rculo, y sea P la proyección


de I sobre AB. Demuestre que 4M 0 CM ' 4AIP .
d) Sea O el circuncentro del triángulo Calcule la potencia de punto de I con respecto al
circuncı́rculo, y utilizando los resultados anteriores deduzca la Fórmula de Euler : OI 2 =
R2 − 2Rr.
e) A partir de la fórmula de Euler demuestre que R ≥ 2r.

UES-FCNM-Escuela de Matemática-Licenciatura en Matemática y Licenciatura en Estadı́stica


pág. 94

S-ar putea să vă placă și